Sie sind auf Seite 1von 87

P pleM

Peop
MakeCh
hoicces
MR
RCPIPaartIIWrittenM
MockTeest

7/11
1/2008
Neu
urology4MRC
CP.com

Dr.OsamaS.M
M.AminMR
RCPIMRCPSS(Glasg)

People Make Choices


MRCPI Part II Written Mock Test with 3 Papers
Best of Many, Case Histories, and Data Interpretation

By

Dr. Osama S.M. Amin


MBChB MRCPI MRCPS(Glasg)

July 2008

Copyright Notice:

All rights reserved. No part of this book may be reproduced without a prior
permission from the author. You may download, forward, or print the book for
individual use only, not for commercial use; otherwise, contact the author.

Disclaimer:
This book was written depending on reliable sources. However, while every effort has
been made to ensure its accuracy, no responsibility for loss, damage, or injury
occasioned on any person acting or refraining from action as a result of information
contained herein can be accepted by the author.

2008. Neurology4MRCP.com . All rights reserved

"I believe ... that every human mind feels pleasure


in doing good another."
Thomas Jefferson

To

MRCPI candidates, current and future


with all the very best

Osama

Preface:

any candidates who have successfully passed the MRCPI part I


examination have queries about the part II written one. They have no
difficulty with the BoM paper, but they are not familiar with the case
histories and data interpretation ones.
MRCPI part II written examination has 3 papers; BoM, case histories, and data
interpretation. These should be finished within a 3-hour period; there is no break.
The BoM usually has 30 questions (may be more or less depending on the diet) and
has the bulk of the final score and that is 150/250. Other papers have 8-10 questions
and may have a score of 5010. Photographic materials (X-ray, CT scan, ECG,
patient's face) can be found in each paper.
I suggest starting with the BoM paper as it has the bulk of the score.
Case histories and data interpretation questions are to be completed with a single
line answer; use a pencil and eraser. Many candidates know the answers but they fail
to translate this into a single line sentence. For example. A question is about mitral
regurgitation's diagnosis in the data interpretation paper, and the candidate answers
"mitral regurgitation" instead of "severe mitral regurgitation with mild pulmonary
hypertension and atrial fibrillation"; note the full diagnosis! Be careful.
I tried to make a full mock test for MRCPI part II written examination to help the
candidates be familiar with its outline.
I'm grateful to Dr. S. Schwanni M.D. for the photographic materials which were
taken during our clinical rounds.
All the very best

Osama Amin
July 10 2008
Dr. Osama S.M. Amin MRCPI MRCPS(Glasg)

Notice:
The Arrangement of the Mock Test is as follows:
1.
2.
3.
4.
5.
6.
7.

Best of Many paper, 30 questions


Case Histories,10 questions
Data Interpretation,10 questions
Photographic materials, 12 in number
BoM paper answers
Case histories answers
Data interpretation answers

page 1
page 17
page 28
page 39
page 53
page 67
page 73

This page is intentional left blank

Dr.Osama S.M Amin MRCPI MRCPS(Glasg)

MRCPI Part II Written Mock Test

Best of Many Paper

eu

ro

lo
gy

4M
R

P.
co

Total 30 Question

[1]

www.Neurology4MRCP.com

Dr.Osama S.M Amin MRCPI MRCPS(Glasg)

MRCPI Part II Written Mock Test

What would you do next to confirm the cause of his chest pain?

P.
co

Treadmill exercise ECG testing.


Coronary angiography.
Dobutamine stress echocardiography.
Myocardial perfusion imaging.
Repeat his 12-lead resting ECG.

4M
R

a.
b.
c.
d.
e.

Q1) A 65-year-old retired police officer presents with central chest pain over the past 5 months.
The patient says that his chest pain occurs when he walks 2 blocks on the flat level, against hill,
and when doing the stairs, that is relieved by rest for about 10 minutes. His blood pressure is
always high as his GP told him and he does not take his oral anti-diabetic medication regularly.
He smokes 2 packets of cigarettes per day, and drinks 3 units of alcohol at weekends only. His
past surgical history reveals right-sided total hip replacement 4 years ago, but there is no family
history of note. His daily medications are lisinopril 5 mg once daily with daily metformin 850
mg. His examination reveals BMI of 28 Kg/m2, regular pulse rate of 90 beats/minutes with a
blood pressure of 170/100 mm Hg. The neck is supple and abdomen is benign. His chest shows
features of COPD with normal precordial auscultation. The left knee is swollen and painful on
movements with a crepitus. His 12-lead resting ECG that was done by his GP last week is
normal-looking. You have ordered some bloods to be done.

eu

ro

lo
gy

Q2) A 61-year-old journalist visits the physicians office with exertional breathlessness. He has
cough with scanty whitish sputum and occasional wheeze. He is life-long heavy smoker. He
denies any form of chest pain. Examination reveals hyper-inflated chest, hyper-resonance
percussion note with diminished breath sounds all over. There are few scattered wheezes. The
liver edge is palpable but there is no splenomegaly. His bloods show hemoglobin of 9 g/dl, serum
potassium 3.0 meq/L, and leuko-erythroblastic blood picture. He takes many forms of daily
inhalers besides oral prednisolone but with little relive of his chest problem. His plain chest film
is consistent with emphysema and there is an irregular rounded opacity, 3 x 2.5 cm, a little above
and lateral to the right hilar region. His up-to-date FEV1 is 50% the predictive value. Abdominal
ultrasound is negative and his brain CT scan is normal. Biopsy of the mass is consistent with
small cell lung cancer.
How would you treat?
a.
b.
c.
d.
e.

Surgical removal of the mass.


Endoscopic ablation of the mass.
Hospice care.
Radiotherapy with chemotherapy.
Radiotherapy.

[2]

www.Neurology4MRCP.com

Dr.Osama S.M Amin MRCPI MRCPS(Glasg)

MRCPI Part II Written Mock Test

Q3) A 30-year-old secretary comes for check-up. She was lastly seen by her GP 4 years ago. She
has type I diabetes mellitus since the age of 14 years, for which she takes insulin injections. She
says that her fingers and feet numb and she feels tired most of the time in spite of having good
sleep at night. Her duties at work are demanding and she misses a meal every now and then with
frequent hypos. She denies chest pain or breathlessness. She neither smokes nor drinks alcohol.
She lives with her husband in a 2-story house but they have no children. Her daily medications
are soluble and isophane insulin and tonics. Examination reveals a thin woman with MBI 20
Kg/m2, pulse rate 86 beats per minutes and blood pressure of 160/105 mm Hg with pitting ankle
edema. There is diminished pinprick and touch sensations in a stocking pattern with planter
callosities. Her fundi have background retinopathy. Her neck, chest, and abdomen are
unremarkable. Serum triglyceride is 310 mg/dl, serum cholesterol is 240 mg/dl, serum LDLcholesterol is 140 mg/dl, and her urine is positive for protein and sugar. Blood urea is 85 mg/dl
and serum creatinine is 1.9 mg/dl. HBA1c is 9%.

Optimize her glycemic control.


Add a statin.
Do renal biopsy.
Prescribe enalapril.
Give gabapentine at night.

4M
R

a.
b.
c.
d.
e.

P.
co

All of the following are part of your management plan, except which one?

eu

ro

lo
gy

Q4) A 43-year-old salesperson has undergone allogenic renal transplantation 4 weeks ago
because of end stage renal disease of unknown cause. He takes daily cyclosporine, azathioprine,
and prednisolone and he was doing well until today morning. Today, he developed 3 generalized
tonic-clonic seizures. His GP has been treating him for flu with clarithromycin and
paracetamol. His flu seems to be improved. His wife denied head trauma or confusion and she
says that her husband is fully compliant with his daily medications. His father died at the age of
63 years because of diabetic nephropathy and his older brother has idiopathic generalized
epilepsy. The patients past records uncover pyogenic meningitis at the age of 6 years and
tonsillectomy at the age of 8 years. Examination reveals no fever or neck stiffness, and the patient
is fully conscious with flexor planters. His blood urea and serum creatinine have been stable over
the past 2 weeks.
What is the likely cause of these fits?
a.
b.
c.
d.
e.

Cerebral toxoplasmosis.
Streptococcal meningitis.
Idiopathic grand mal epilepsy.
Cyclosporine toxicity.
Uremic encephalopathy.

[3]

www.Neurology4MRCP.com

Dr.Osama S.M Amin MRCPI MRCPS(Glasg)

MRCPI Part II Written Mock Test

Q5) A 58-year-old carpenter presents with fatigue for 6 months. He says that he is unable to do
his job as he has no power to work, which forced him to supervise the work only. His wife says
that he eats and sleeps well but he is obviously pale. He denies a change in his bowel habit and
there is no weight loss but he has dyspepsia ascribed to irritable bowel syndrome as his GP told
him. He is being treated for stage I hypertension with chlorthalidone and antispasmotics for his
irritable bowel. His father died of colonic cancer when he was 70 years and his mother died of
massive stroke at the age of 75 years. His only sister has Crohns disease. He smokes 2-4
cigarettes a day and drinks a glass of wine every night. His GP has ordered colonoscopy for him
which is then turned to be normal. His hemoglobin is 6.7 g/dl and the RBCs are hypochromic
microcytic but the WBCs and platelets are normal. His blood urea and electrolytes are within
their normal reference range as are his liver parameters. Serum ferritin is 40 mol/L.

P.
co

Repeat colonoscopy.
Refer for upper GIT endoscopy.
Do sigmoidoscopy.
Order fecal occult blood testing.
Bone marrow study.

4M
R

a.
b.
c.
d.
e.

What would you do to elucidate the cause of his fatigue?

eu

ro

lo
gy

Q6) A 32-year-old high school teacher presents with subacute loss of right eye vision. She says
that her right eye became blurred a week ago and the globe movements are painful and the
blurring gradually increased to a degree that she can not see anything clearly with her right eye.
She denies headache or orbital trauma. She takes oral contraceptive pills for the past 2 years and
ibuprofen for occasional backpain. Review of systems is unremarkable as is her past medical and
surgical histories; however, she admits to trying amphetamine 10 years ago, once. Her older sister
has diabetes and her mother has hypertension and chronic glaucoma. Examination of the right eye
reveals diminished visual acuity to 6/60, Marcus-Gunn pupil, and normal-looking fundus. The
rest of her neurological examination is normal as is her medical one. Brain CT scan is normal but
her brain MRI shows many periventricular oval-shaped plaques. CSF examination is normal and
her bloods are within their normal reference range. Urinary and blood toxicology screen is
negative. Visual evoked response of the left eye is normal. You started methylprednisolone
infusions over 5 days and she made an excellent improvement after 2 weeks.
How would you manage her in the long-term?
a.
b.
c.
d.
e.

Daily oral prednisolone.


Daily azathioprine.
Monthly methylprednisolone pulses.
Wait and see.
Beta interferon therapy.

[4]

www.Neurology4MRCP.com

Dr.Osama S.M Amin MRCPI MRCPS(Glasg)

MRCPI Part II Written Mock Test

Q7) A 41-year-old housewife has been diagnosed with rheumatoid arthritis 3 years ago and she
takes daily sulphasalazine and weekly methrotrexate. Today, she is visiting you with a concern
about her eyes. She says that her eyes burn and she feels as if there is sand in them. Her hands are
painful on movements with mild ulnar deviation of the fingers. Her rheumatoid factor titer is
positive and there is normochromic normocytic anemia with raised ESR and thrombocytosis.
What is the best treatment for her ocular complaints?
Give methotrexate daily.
Daily oral prednisolone.
Monthly gold injections.
Methylcellulose eye drops.
Topical dexamethasone drops.

a.
b.
c.
d.
e.

eu

ro

Decrease the daily L-thyroxin.


Add growth hormone.
Increase the daily L-thyroxin.
Keep the same daily dose of L-thyroxin.
Add cortisone.

a.
b.
c.
d.
e.

lo
gy

What would you do next?

4M
R

P.
co

Q8) A 34-year-old TV director comes to the physicians office because of her thyroid problem.
She has been diagnosed with hypothyroidism 1 year ago because of Hashimotos thyroiditis. Her
only current medication is L-thyroxin 75 g per day over the past 8 months. She denies weight
gain, constipation, aches and pains, or menstrual irregularities. Her mother has primary atrophic
hypothyroidism and her father has diabetes and hypertension. She is active at work and she is
planning to direct a new TV series. Her serum T4 is 80 nmol/L (normal 60-145 nmol/L) and her
serum TSH is 15 mU/L (normal 0.5-5.0 mU/L). Examination shows pulse rate of 70 beats per
minutes and blood pressure of 110/85 mm Hg.

[5]

www.Neurology4MRCP.com

Dr.Osama S.M Amin MRCPI MRCPS(Glasg)

MRCPI Part II Written Mock Test

Q9) A 41-year-old biologist is referred by your psychiatry colleague. The referral states that the
patient is being treated for chronic schizophrenia but recently he has had jaundice. The patient
says that he is compliant with his daily chlorpromazine tablets. He also reports itching. He did not
answer the question as to whether the color of urine and stool has changed, because he did not
watch his excreta as he said. His schizophrenia has been fluctuating over the last 10 years. He
denies nausea, vomiting, abdominal pain, or any change in his bowel habit. He drinks 2 cans of
beer every other day, and smokes 3 cigars daily. He displays no risk factors for HIV infection.
His mother has affective psychosis. Examination reveals yellowish discoloration of the sclera
with generalized hyperpigmentation and excoriation marks. Hepatitides screen reveals positive
serum IgG anti-HBs. Total serum bilirubin is 4 mg/dl, the indirect serum bilirubin is 1 mg/dl,
serum alkaline phosphatase is 200 iu/l while serum AST and ALT are 35 iu/L and 34 iu/L
respectively.

P.
co

Alcoholic hepatitis.
Non-alcoholic liver macrosteatosis.
Chlorpromazine-induced cholestatic hepatitis.
Chronic hepatitis B infection.
Primary biliary cirrhosis.

4M
R

a.
b.
c.
d.
e.

What is the explanation for the current complaint?

eu

ro

lo
gy

Q10) A 45-year-old homosexual airplane pilot presents with fever and skin rash for the last 1
week. He has oral candidiasis and palpable neck lymph glands. The pharynx is congested and
there is neck stiffness. He admits to having protected sex with men during his trips. His current
daily medications are amlodipine for hypertension and simvastatin for hypercholesterolemia. He
denies vomiting, weight loss, or breathlessness. After careful counseling, he agreed to undergo
HIV testing. His plasma HIV RNA viral load is 210000 copies/ml and the CD4+ count is 260
cells/ml3.

a.
b.
c.
d.
e.

All of the following statements are wrong in respect to his current diagnosis, except which one?
A prolonged illness indicates a rapid progression to AIDs.
Oral ulceration is against the diagnosis.
Atypical lymphocytes in blood indicate co-existent EBV infection.
Neck stiffness is due to cryptococcal meningitis.
There is low infectivity rate.

[6]

www.Neurology4MRCP.com

Dr.Osama S.M Amin MRCPI MRCPS(Glasg)

MRCPI Part II Written Mock Test

Q11) A 32-year-old taxi driver is brought to the ER by his wife short of breath. He has chronic
persistent asthma for the last 6 years and is compliant with his daily glucocorticoid and beta
agonist inhalers. His wife says that he has had a flu and his GP prescribed paracitamol and
ampicilline for him 3 days ago but today his breathlessness has increased. He is afebrile and
conscious and his pulse is regular at a rate of 100 beats/minutes, blood pressure 110/60 mm Hg,
and his chest is wheezy but there are no crackles. Chest X-ray film shows generalized hyperlucent shadow with no pneumonic patch or pneumothorax. Blood gas analysis on air is as
follows: PO2 8.5 kPa, PCO2 4.0 kPa, PH not done. He is being given high flow high concentration
oxygen.

P.
co

High dose intravenous hydrocortisone.


Montelukast.
Intravenous magnesium.
Mechanical ventilation.
Chest tube insertion underwater seal apparatus.

a.
b.
c.
d.
e.

What is the best action to be taken next?

eu

ro

lo
gy

4M
R

Q12) A 69-year-old retired dietician presents to the ER with central chest pain for the last 6 hours
that is partially responsive to many sublingual nitroglycerin. The patients sister says that her
brothers pain built-up gradually while he was reading the newspaper and he decided to visit the
ER after 6 hours for fear of a cardiac attack. He had 3 short-lived anginal attacks within the past
24 hours. He has chronic stable angina, hypertension, hypercholesterolemia, and type II diabetes.
He smokes about 10 cigarettes per day but does not drink alcohol. His father died of myocardial
infarction a the age of 71 years. His GP prescribed atenolol, aspirin, and isosorbide dinitrate. A
12-lead ECG shows deep symmetrical T-wave inversion in the precordial leads from V1 to V6 but
there is no Q-wave. Serum troponin-I is minimally raised. The plain chest film is unremarkable
as are his bloods. Precordial examination was unhelpful and his blood pressure is 150/95 mm Hg.
He has received morphine, aspirin, clopidogrel, heparin, metoprolol, abciximab, and nitroglycerin
infusion and he is now stabilized.
Which one of the following actions do you consider to be the most appropriate to be applied for
the time being?
a.
b.
c.
d.
e.

Intravenous alteplase.
Exercise ECG stress testing.
Coronary angiography and revascularization.
Intra-arterial lepirudin.
Oral ramipril.

[7]

www.Neurology4MRCP.com

Dr.Osama S.M Amin MRCPI MRCPS(Glasg)

MRCPI Part II Written Mock Test

Q13) A 25-year-old sewage worker presents with sore throat and enlarged tender cervical lymph
nodes. Oral ampicilline was started and the patients pharyngitis started to resolve. Today, he is
visiting the physicians office because of fever, skin rash, and joint pains. His past medical
history is notable for gonorrhea 2 years ago and typhoid fever 9 months ago. There is no family
history of note. He denies chest pain, bowel symptoms, or dysuria, and he says that his urine
output is maintained. His bloods reveal leukocytosis and eosinophil count 10%, mildly raised
blood urea and serum creatinine. His urine is positive for protein, blood, and eosinophils.

Acute glomerulonephritis.
Disseminated gonococcal disease.
Epstein-Barr viral infection.
Hanta viral infection.
Drug-induced acute allergic interstitial nephritis.

P.
co

a.
b.
c.
d.
e.

What is the cause of the new presentation?

lo
gy

4M
R

Q14) A 54-year-old pharmacist visits the physicians office because of lethargy. He says that he
can not cope with his drugstore work and this lethargy has been increasing over the last 9 months.
He has lost interest in sex, and does not go to his favorite local diner at weekends as he used to
do. He reports nausea, loss of appetite, and constipation. He feels dizzy when he stands suddenly
from a low chair. His past medical history shows successfully treated post-primary tuberculosis 1
year ago. Examination reveals a man with a downcast face, tanned, thin, and with postural
hypotension. His blood urea is 55 mg/dl, serum potassium is 5.5 meq/L, and his blood sugar is 70
mg/dl.
What is the best investigation to start with to confirm your clinical suspicion?

eu

ro

Complete blood count.


Serum calcium.
Morning serum ACTH and cortisol.
Serum growth hormone during sleep.
Renal biopsy.

a.
b.
c.
d.
e.

[8]

www.Neurology4MRCP.com

Dr.Osama S.M Amin MRCPI MRCPS(Glasg)

MRCPI Part II Written Mock Test

Q15) A 55-year-old office manager comes for his annual check-up visit. He is reasonably well
and healthy, and has no chronic diseases. He lives alone in an apartment, does the shopping, and
jogs every Sunday morning in the local park. He does not drink but does smoke a cigarette or two
every day. His past medical and surgical histories are unremarkable as is his family history.
Examination reveals no organomegaly or lymph node enlargement. Abdominal ultrasound did not
show any liver, spleen, or lymph node enlargement. His complete blood count shows hemoglobin
14 g/dl, white cells 25 x 10 9/L with 95% mature-looking small lymphocytes, and platelets count
200 x 109/L.

P.
co

He has stage I disease.


He should receive monthly IVIG infusions.
The best treatment is observation.
There is no increased risk of developing solid malignancy.
Short doubling time carries good prognosis.

a.
b.
c.
d.
e.

Which one of the following is true regarding this mans disease?

eu

ro

lo
gy

4M
R

Q16) A 42-year-old factory worker presents with hand pain and stiffness for the last 5 months. He
says that his hands sore and are painful when using them and they are especially stiff upon
awaking in the morning. He is not that bothered by his low back pain as his GP told him that it is
a form of mechanical one. He denies neck pain, chest pain, bowel symptoms, and urinary
complaints. He drinks a little whisky every night but does not drink or do drugs. His older brother
has a rigidity problem in his entire spine and has received a treatment for it for 15 years. He has
mild intermittent asthma which is responsive to inhalers. He takes an injection of vitamin B12
every month as he thinks that this is a healthy habit. Examination reveals swelling of the distal
interphalangeal joints of both hands with painful active and passive movements with limited
spinal movements and tenderness of the lower back. Large proximal joints are normal-looking.
His ESR is 60 mm/hour.
What does the man have?
a.
b.
c.
d.
e.

Ankylosing spodylitits.
Reiters syndrome.
Rheumatoid arthritis.
Adult Stills disease.
Psoriatic arthropathy.

[9]

www.Neurology4MRCP.com

Dr.Osama S.M Amin MRCPI MRCPS(Glasg)

MRCPI Part II Written Mock Test

Q17) A 61-year-old retired typewriter presents with an 8-month history of clumsy hands and
difficulty in walking. He says that he has lost his manual dexterity and has some sort of difficulty
using the cutlery and working on the computer, and his legs are becoming unable to hold his body
when standing or walking. Besides, his hands and feet numb all the time and sometimes he feels
as if there are worms marching on his feet. He denies any problem with swallowing or control of
his urine or bowel. He underwent 2 cataract surgeries last year and he has no double vision. He is
being treated for NYHA functional class II congestive heart failure and his diabetes is unstable
for which he has declined the idea of receiving insulin injections. He is compliant with aledronate
tablets after sustaining an osteoporotic hip fracture 3 years ago. He neither smokes nor drinks
alcohol. His older brother has had ischemic stroke. Examination reveals wasting of hands and feet
and with diminished pinprick, touch, and joint position sensations. There is grade 4 minus
weakness, proximal and distal, in both upper and lower limbs with absent deep tendon reflexes.
Cranial nerves are intact and there is no sensory level. CSF protein is 200 mg/dl, and there is
prolonged F-wave in upper and lower limbs with prolonged distal latency.

Prednisolone.
Plasma exchange.
IVIG.
Riluzole.
-interferon 1a.

4M
R

a.
b.
c.
d.
e.

P.
co

What is the best treatment this man should receive for his presenting illness?

eu

ro

lo
gy

Q18) A 32-yea-old primary school janitor comes for his scheduled check-up. He has been given a
diagnosis of duodenal ulcer 4 weeks ago after doing upper GIT endoscopy which showed 2x2 cm
duodenal bulb ulcer but without taking a biopsy, and treatment was started at that time with triple
therapy consisting of lansoprazole, amoxilline, and clarithromycin. Today, he says that he has the
same initial symptoms of epigastric hunger pain that is relieved by food. He denies any
medication non-compliance. He does not smoke or drink alcohol. His mother died of esophageal
cancer 2 years ago at the age of 65 years. His older brother underwent gastric surgery for gastroesophageal reflux disease.
How would you respond?
a.
b.
c.
d.
e.

Repeat esophagogastroscopy.
Barium meal.
Barium follow through.
Repeat the same treatment for another 4 weeks.
Urea breath testing.

[10]

www.Neurology4MRCP.com

Dr.Osama S.M Amin MRCPI MRCPS(Glasg)

MRCPI Part II Written Mock Test

Q19) A 39-year-old male PhD student presents with fever and skin rash for 5 days. He returned
form Thailand 1 week ago and spent 2months there as part of students exchange program. He also
reports retro-orbital pain, severe bone and muscle pains, with nausea and vomiting. He does not
remember any insect bite and he declined being vaccinated before travelling 2 moths ago. He
took no prophylactic medications for infections and has had no sex there. As far as he knows, he
has not come in contact with sick people. He takes daily sodium valproate for migraine
prophylaxis and he denies doing drugs. His sister has common variable immune deficiency.
Examination reveals fully conscious patient, temperature 39.1 Co, diffuse maculopapular rash,
conjunctival injection, pharyngeal redness, and palpable liver edge. His bloods show leukopenia,
thrombocytopenia, and mildly raised serum AST.
What does the man have developed?

P.
co

Yellow fever.
Classic dengue fever.
Crimean-Congo hemorrhagic fever.
Dengue hemorrhagic fever.
Acute HIV seroconversion illness.

a.
b.
c.
d.
e.

eu

ro

lo
gy

4M
R

Q20) A 31-year-old woman visits the physicians office because of poor exercise tolerance. She
was diagnosed with NYHA functional class III idiopathic dilated cardiomyopathy 7 months ago
after having breathlessness and orthopnea with raised JVP, bibasal crackles, and leg edema.
Currently she receives enalapril 10 mg twice daily, carvedilol 12.5 mg twice daily, spironolactone
25mg once daily, and frusamide 20 mg once daily. She is compliant with her regimen and she
denies the ingestion of any other medication or doing drugs. Specific enquiry about her current
complaint has the answer of I feel weak when walking on the flat after some time, doing the
stairs, scrubbing or mopping the floor, or after dusting the furniture home. Examination reveals
mildly raised JVP and trace leg edema, clear lung bases and no gallop rhythm. Her ECG has
wide-spread non-specific ST-T changes with QRS complex duration of 17 msec. Up-to-date
ejection fraction is 34%. Her blood pressure is 100/60 mm Hg, and pulse rate 62 beats / minute
that is regular in rhythm and volume.
What would you do to improve her exercise tolerance?
a.
b.
c.
d.
e.

Give amiodarone.
Start digoxin.
Arrange for resynchronization therapy.
Refer for cardiac transplantation.
Increase the dose of diuretics.

[11]

www.Neurology4MRCP.com

Dr.Osama S.M Amin MRCPI MRCPS(Glasg)

MRCPI Part II Written Mock Test

Q21) A 6-year-old boy is brought by his mother to your office. She says that her son has had
nephrotic syndrome that was diagnosed 3 months ago after doing some investigations of blood
and urine. She insists that her sons face is still puffy and his legs are swollen with little
improvement since his treatment has been initiated with prednisolone 60 mg a day. She gives the
medication to him every day and has never missed a dose. She declined insect bites or allergies.
The childs uncle died of polycystic kidney disease and his grandmother has diabetic
nephropathy. The boys mother is anxious and desperate for your help. Examination reveals gross
Cushingoid habitus, periorbital puffiness, 3 plus pitting leg and sacral edema. His blood pressure
is 150/100 mm Hg; it was 110/60 mm Hg at the time of the diagnosis was made. His urine shows
4 plus proteinuria, but with no blood or casts. Serum cholesterol is raised and there is
hypogammaglobulinemia.

P.
co

Increase the dose of prednisolone.


Continue the same dose for another 3 months.
Add cyclphosphamide.
Do renal biopsy.
Refer for kidney transplantation.

4M
R

a.
b.
c.
d.
e.

What would you do next?

eu

ro

lo
gy

Q22) A 26-year-old male presents to the ER breathless. His girlfriend says that they were
watching a TV talk show when he experienced sudden precordial chest pain with little shortness
of breath and that he ascribed this to his exhausting daily job as clerk in the stock market. Then,
she said, his pain has increased and the breathless is building-up itself to a marked degree. He
smokes 1 packet of cigarettes per day and drinks few glasses of burgundy every few days at night.
He denies doing drugs and there are no risk factors for HIV. His family history is unremarkable.
Examination reveals a young man with obvious distress and breathlessness, blood pressure 80/40
mm Hg, pulse rate 130/minutes regular, and hyper-resonance percussion note on the left
hemithorax. The chest film is of a poor quality due to patients uncooperativeness. A chest tube
was inserted under water seal apparatus on the left side and air went out. However, he is still
unwell.
What do you think that it is the appropriate thing to do now?
a.
b.
c.
d.
e.

Insert another chest tube on right side.


Chest aspiration of the left pleural space.
Apply suction on the inserted chest tube.
Give high flow high concentration oxygen.
Do 12-lead ECG.

[12]

www.Neurology4MRCP.com

Dr.Osama S.M Amin MRCPI MRCPS(Glasg)

MRCPI Part II Written Mock Test

Q23) A 45-year-old man is referred to by his GP as a newly diagnosed case of acromegaly. The
patient is hypertensive and has bilateral carpal tunnel syndrome. His head seems to be large. He
has been subjected to many lab tests and his bloods show normal blood counts and ESR,
hyperglycemia and glycosuria, normal renal function, and insulin-like growth factor-I level in the
lower part of its reference range. His GP has already arranged measurement of serum growth
hormone and brain MRI.
What is your response?
Start pegvisomant.
Discuss the matter with the neurosurgeon.
Take the opinion of the anesthetist for fitness under general anesthesia.
He has no acromegaly.
Do insulin tolerance test.

P.
co

a.
b.
c.
d.
e.

lo
gy

4M
R

Q24) A 72-year-old poet visits the physicians office. The man says that he has been diagnosed
with Pagets disease of the bone 2 years ago after having pain in his mid-spine with enlarged D10
and D11 vertebrae. His back pain is responsive to mefenamic acid and he is not that bothered by
that pain. He is house-bound and spends most of his time in bed reading. He denies headache or
hearing problem and his legs does not seem to be bowed. His blood pressure is well controlled
with alpha methyldopa and his blood lipids are responding to atrovastatin. Blood counts are
normal as are his renal function tests with a serum calcium of 11.8 mg/dl. Serum alkaline
phosphatase is twice its upper normal limit.
What is the correct statement?

eu

ro

He needs spinal fixation surgery.


Prednisolone is better to be given.
No treatment is needed.
Bone scanning should be done.
Start oral tiludronate.

a.
b.
c.
d.
e.

[13]

www.Neurology4MRCP.com

Dr.Osama S.M Amin MRCPI MRCPS(Glasg)

MRCPI Part II Written Mock Test

Q25) A 34-year-old man is referred to you from the hepatology department for further
management. The referral states that the patient is HBs antigen positive that was detected as part
of medical insurance examination. He is an immigrant from Asia. He does not remember being
jaundiced. He practices sex with men and does intravenous drugs. He denies weight loss, itching,
nausea, or malaise. He lives alone, does drink a unit of alcohol at night, and smokes a packet of
cigarettes every day. Examination failed to show hepatomegaly, ascites, or jaundice. His bloods
show negative HBe antigen, elevated plasma HBV DNA, and persistent elevation in serum
aminotransferases. HIV testing is negative as is the testing for hepatitis C virus. Serum albumin
and PT are normal.

P.
co

Hepatocellular carcinoma.
Liver cirrhosis.
Chronic hepatitis B infection with a pre-core mutant form.
Chronic hepatitis C infection.
HIV induced peliosis hepatis.

a.
b.
c.
d.
e.

What does the man have?

lo
gy

4M
R

Q26) A 42-year-old singer presents with bilateral hand tremor. The tremor has been present since
the age of 20 years and is somewhat stable form that time. He denies sustained contractions of
limbs, swallowing difficulty, frequent falls, or jaundice. His father has the same problem. He does
not smoke but he does occasionally drink alcohol. Examination reveals postural tremor in both
hands, with normal power, intact deep tendon reflexes, and flexor planters. Sensation is intact as
is the sensorium. He takes regular beclomethasone and salmetrol inhalers for asthma. He asks for
your help and he is concerned about his embarrassing appearance.

eu

Botulinum toxin.
Reassurance.
Propranolol.
Primidone.
Alprazolam.

a.
b.
c.
d.
e.

ro

What is the best way to treat this man?

[14]

www.Neurology4MRCP.com

Dr.Osama S.M Amin MRCPI MRCPS(Glasg)

MRCPI Part II Written Mock Test

Q27) A 19-year-old female is referred by her GP because of an itchy skin condition. The patient
has hyperpigmented lichenified papular rash over the face, antecubital and popliteal fossae with
excoriation marks but little oozing and scaling. She has had this rash since the age of 1 year but
its sites are changing every few years. Her older brother has intermittent asthma. She is currently
sexually inactive, and her menstrual cycle is irregular. Her only medication is daily paracetamol
as she thinks that will prevent the stress of the coffee shop she works in. Her HIV status is
negative which was requested by her one year ago after having unprotected sex with a man whom
she met in the coffee shop.
Which one of the following is true in respect to this womans illness?

Progression to wide-spread flaccid skin bullae is common.


Topical tacrolimus should be used for the facial rash.
Superinfections of the rash induce remission.
Family history of the same rash is unusual.
Systemic glucocorticoids are the first line agents in the treatment.

P.
co

a.
b.
c.
d.
e.

ro

lo
gy

4M
R

Q28) A 31-year-old widow is brought to the ER by her older sister. The patient is confused,
tachycardic, and flushed. Her sister says that the patient has been treated for low mood with a
medication since the death of her husband 3 months ago and she (the patient) kept saying that she
will meet him soon. Two hours ago, she found her on her bed irritable and many tablets of her
medication were on the floor. She took her to a local hospital which has referred the patient to
you. Examination reveals a confused woman, with sinus tachycardia, blood pressure 90/50 mm
Hg, flushing, dry skin, dilated pupils and hyperactive reflexes. Resuscitation has begun with
decontamination. Her ECG shows regular heart rate of 140 beats/minute with QRS complex
duration of 160 msec. Normal saline and sodium bicarbonate infusions are being given. Two hour
later, her blood pressure is 80/55 mm Hg and her QRS complex duration is 120 msec.

Mechanical ventilation.
Arrange for hemodialysis.
Give Epicac.
Infuse magnesium.
Noradrenalin infusion.

a.
b.
c.
d.
e.

eu

Choose the correct action for the time being?

[15]

www.Neurology4MRCP.com

Dr.Osama S.M Amin MRCPI MRCPS(Glasg)

MRCPI Part II Written Mock Test

Q29) A 41-year-old female presents with pain all over her body. She says that her spine, limbs,
and chest are painful for the last 2 weeks which are some what alleviated by paracetamol tablets.
One year ago she underwent lumpectomy with localized radiotherapy for stage II right-sided
breast cancer. Shes been doing well until 2 weeks ago when she started to feel out of power with
pains and aches. She is on no regular medication for the time being. Examination of the breasts
shows only the scar of previous surgery and there are no palpable axillary lymph nodes. Serum
alkaline phosophatase is raised and there is hypercalcemia and leukoerythroblastic blood picture.
Many liver target lesions were found on abdominal ultrasonographic examination. Per cutaneous
biopsy of the hepatic lesions reveals breast secondaries which are estrogen receptor negative with
very low level of HER2. Both lung fields were bombarded with cannon balls. She is willing to
receive any treatment and is desperate for help. She keeps saying I dont want to die.

P.
co
C
4M
R

Cranial irradiation.
Bone marrow transplantation.
Partial hepatic resection.
Zoledronic acid.
Breast radiotherapy.
Radical mastectomy.
Trastuzumab.
Chemotherapy.
Pulmonary Irradiation.
Anastrazole.

lo
gy

a.
b.
c.
d.
e.
f.
g.
h.
i.
j.

Choose 2 options as part of this womans treatment plan?

eu

ro

Q30) A 50-year-old woman comes to see you with a problem of having fatigue most of the time.
She also says that her neck, shoulders, axial spine, and limbs are painful. She denies any joint
swelling but she admits to having recurrent colicky abdominal pain with alternating diarrhea and
constipation besides urinary frequency. She insists that she was healthy and had no chronic
illnesses and enjoys independent life. Her sleep is fragmented and her mood is low. She neither
smokes nor drinks alcohol. She is single and has no children. Her GP ascribed these features to be
part of her cessation of menstruation. You are thinking of fibromyalgia.
From the following list, choose 2 options that are inconsistent with your diagnosis?
a.
b.
c.
d.
e.
f.
g.
h.
i.
j.

ESR 50 mm/hour.
TSH 3.0 mU/L.
Hemoglobin 13 g/dl.
Serum alkaline phosphatase 70 iu/L.
Negative ANA.
Normal chest X-ray.
Serum ALT 15 iu/L.
Eosinophil count 2%.
Distended colonic bowel loops on abdominal ultrasound.
Platelets 90 x 109/L.

[16]

www.Neurology4MRCP.com

Dr.Osama S.M Amin MRCPI MRCPS(Glasg)

MRCPI Part II Written Mock Test

Case Histories

eu

ro

lo
gy

4M
R

P.
co

Ten Questions

[17]

www.Neurology4MRCP.com

Dr.Osama S.M Amin MRCPI MRCPS(Glasg)

MRCPI Part II Written Mock Test

Q1) A 17-year-old female presents with short-lived palpitations associated with presyncope. She
has pansystolic murmur at the left lower sternal border that increases with inspiration. Her mother
has bipolar disorder. This is her 12-lead resting ECG (figures 1, 2, 3, and 4).

a. What is the full diagnosis?


-------------------------------------------------------------------------------------------------b. What is the risk of having a similarly affected sibling?
--------------------------------------------------------------------------------------------------

P.
co

c. One year later, after giving birth to a full term girl by 1 week, she presented with this
brain CT scan (figure 5). Why?

eu

ro

lo
gy

4M
R

--------------------------------------------------------------------------------------------------

[18]

www.Neurology4MRCP.com

Dr.Osama S.M Amin MRCPI MRCPS(Glasg)

MRCPI Part II Written Mock Test

Q2) A 40-year-old male presents with burning micturition. Examination shows painless oral
ulcers. This is him (figure 5).

a. What other symptom he might have that is helpful to you?


-------------------------------------------------------------------------------b. Suggest a diagnosis?
--------------------------------------------------------------------------------c. Name 2 skin manifestations?

---------------------------------------------------------------------------------

eu

ro

lo
gy

4M
R

P.
co

----------------------------------------------------------------------------------

[19]

www.Neurology4MRCP.com

Dr.Osama S.M Amin MRCPI MRCPS(Glasg)

MRCPI Part II Written Mock Test

Q3) A 63-year-old man is brought to the ER confused. This is his non-contrast brain CT scan
(figure 6).

a. Give 2 diagnoses this man has?


----------------------------------------------------------------------------------------------------------------------------------------------------------b. Which one of the above diagnoses precedes than the other one?
------------------------------------------------------------------------------

d. Suggest one risk factor for each of the mentioned diagnoses?

P.
co

---------------------------------------------------------------------------------

eu

ro

lo
gy

4M
R

----------------------------------------------------------------------------------

[20]

www.Neurology4MRCP.com

Dr.Osama S.M Amin MRCPI MRCPS(Glasg)

MRCPI Part II Written Mock Test

Q4) A 16-year-old girl is referred to you with fever and right upper abdominal pain. This is her
chest plain film (figure 7).
a. What is the cause of her abdominal symptom?
--------------------------------------------------------------------------------------b. Name 3 other investigations?
----------------------------------------------------------------------------------------------------------------------------------------------------------------------------------------------------------------------------------------------------------------------

P.
co

c. Two week later, she came with fever and chest tightness. Suggest 2 possibilities for the
new presentation?
-----------------------------------------------------------------------------------------

eu

ro

lo
gy

4M
R

-----------------------------------------------------------------------------------------

[21]

www.Neurology4MRCP.com

Dr.Osama S.M Amin MRCPI MRCPS(Glasg)

MRCPI Part II Written Mock Test

Q5) A 25-year-old single woman presents with anxiety, fine postural tremor, and palpitations.
This is her neck (figure 8).

a. What does the woman have?


---------------------------------------------------------------------b. What mode of treatment she has had in the past?
-----------------------------------------------------------------------

eu

ro

lo
gy

4M
R

P.
co

-----------------------------------------------------------------------

c. How would you treat now?

[22]

www.Neurology4MRCP.com

Dr.Osama S.M Amin MRCPI MRCPS(Glasg)

MRCPI Part II Written Mock Test

Q6) A 51-year-old man presents with recurrent hypoglycemia and polyuria. Examination shows
band-shaped keratopathy. This is his left hand (figure 9).
a. What is the overall diagnosis?
---------------------------------------------------------------------------------------------b. Suggest 2 causes for his polyuria?
------------------------------------------------------------------------------------------------------------------------------------------------------------------------------------------c. Is family history relevant? Why?

eu

ro

lo
gy

4M
R

P.
co

----------------------------------------------------------------------------------------------

[23]

www.Neurology4MRCP.com

Dr.Osama S.M Amin MRCPI MRCPS(Glasg)

MRCPI Part II Written Mock Test

Q7) A 60-year-old refugee presents with bilateral knee pain, dropped beats, and headache. This is
her (figure 10). She has a history of transient skin rash in the form of enlarging erythematous
lesion with central clearing o the back of the knee.

a. Suggest a diagnosis?
------------------------------------------------------------------------------b. Name 3 useful investigations?
--------------------------------------------------------------------------------

--------------------------------------------------------------------------------

c.

P.
co

-------------------------------------------------------------------------------What treatment she should receive?

--------------------------------------------------------------------------------

4M
R

d. Name 2 infections that can be transmitted by the same vector?


--------------------------------------------------------------------------------

eu

ro

lo
gy

--------------------------------------------------------------------------------

[24]

www.Neurology4MRCP.com

Dr.Osama S.M Amin MRCPI MRCPS(Glasg)

MRCPI Part II Written Mock Test

Q8) A 62-year-old man presents with blurred vision. This is his hands X-ray film (figure 11).
a. Name 2 iatrogenic causes for visual impairment?
--------------------------------------------------------------------------------------------------------------------------------------------b. Have a look at his left arm (figure 12). Enumerate 2 findings?
-----------------------------------------------------------------------------------------------------------------------------------------------

P.
co

-----------------------------------------------------------------------------------------------------------------------------------------------

d. His planters are up. Why?

eu

ro

lo
gy

4M
R

--------------------------------------------------------------------------

[25]

www.Neurology4MRCP.com

c. His spleen is palpable. Name 2 causes?

Dr.Osama S.M Amin MRCPI MRCPS(Glasg)

MRCPI Part II Written Mock Test

Q9) A 48-year-old man has ulcerative colitis for 12 years. Today he presents with right upper
abdominal discomfort and raised serum alkaline phsophatase but no fever.
a. Name 2 possibilities for the current complaints?
------------------------------------------------------------------------------------------------------------------------------------------b. What investigations would you order? Name 4?
----------------------------------------------------------------------

----------------------------------------------------------------------

eu

ro

lo
gy

4M
R

P.
co

----------------------------------------------------------------------

----------------------------------------------------------------------

[26]

www.Neurology4MRCP.com

Dr.Osama S.M Amin MRCPI MRCPS(Glasg)

MRCPI Part II Written Mock Test

Q10) A 54-year-old nonsmoker man has allergic rhinitis for 2 years. His chest is wheezy and his
distal limbs numb. ESR is 90 mm/hour.
a. What is the likely diagnosis?
-----------------------------------------------------------------b. Later on he developed pitting led edema. List 2 causes?
-----------------------------------------------------------------------------------------------------------------------------------

P.
co

------------------------------------------------------------------

c. What would his blood counts show? Name 4 findings?

------------------------------------------------------------------

------------------------------------------------------------------

d. What is the usual treatment?

4M
R

------------------------------------------------------------------

eu

ro

lo
gy

------------------------------------------------------------------

[27]

www.Neurology4MRCP.com

Dr.Osama S.M Amin MRCPI MRCPS(Glasg)

MRCPI Part II Written Mock Test

Data Interpretation

eu

ro

lo
gy

4M
R

P.
co

Ten Question

[28]

www.Neurology4MRCP.com

Dr.Osama S.M Amin MRCPI MRCPS(Glasg)

MRCPI Part II Written Mock Test

1) A 32-year-old woman presents with progressive exercise intolerance. These are her cardiac
catheterization study results.
Pressure (mm Hg)
15
89/25
105/40
24
110/3
112/71

RA (mean)
RV
PA
LA (mean)
LV
Aorta

Saturation (%)
75
75
76
94
94
-

a. Enumerate 4 findings?

-------------------------------------------------------------

4M
R

b. What is the full diagnosis?

-------------------------------------------------------------

P.
co

-------------------------------------------------------------

-------------------------------------------------------------

-----------------------------------------------------------------------------------------------------------------

lo
gy

c. What is the likely cause?

eu

ro

--------------------------------------------------------------

[29]

www.Neurology4MRCP.com

Dr.Osama S.M Amin MRCPI MRCPS(Glasg)

MRCPI Part II Written Mock Test

Q2) A 43-year-old man is referred to the respiratory clinic for further evaluation of shortness of
breath. He had bilateral hemothoraces 2 years ago after a road traffic accident. You have ordered
pulmonary function testing and these are the results:
Patients Results
2.4
3.2
75
7.01
1.9

FEV1 (L)
FVC (L)
FEV1/FVC %
DLCO (mmol/min/kPa)
KCO (mmol/min/kPa/L)

Predicted Value
4.18
4.80
78
10.27
1.74

% of Predicted
57
66
68
109

a. Interpret the above results?

b. Suggest the cause for his shortness of breath?

P.
co

-----------------------------------------------------------------------------------------------------------------------------------------------------------------------------------------------------------------------------------------

eu

ro

lo
gy

4M
R

-----------------------------------------------------------------------

[30]

www.Neurology4MRCP.com

Dr.Osama S.M Amin MRCPI MRCPS(Glasg)

MRCPI Part II Written Mock Test

Q3) A 21-year-old man presents with progressive unintentional weight loss with bulky offensive
stool. These are his investigations:

70 mmol (normal <18)


Negative
Undigested food particles, no parasites, blood or puss.
Negative
Negative
Positive
Subtotal villous atrophy
110 fL
Hyochromic RBC
34 mg/dl
3.3 meq/L
14 iu/L

a. What is the diagnosis?


-------------------------------------------------------------

P.
co

Fecal fat
Fecal occult blood
Stool microscopy
Fecal culture
IgA anti-endomysium
IgG anti-tissue transglutaminase
Duodenal biopsy
MCV
Blood film
Blood urea
Serum potassium
AST

b. What is the cause of his negative testing for IgA anti-endomycium antibodies?

4M
R

-------------------------------------------------------------

c. What is the best treatment this man should have?

lo
gy

--------------------------------------------------------------

ro

After 2 months he came back saying that his complaints were improved for only 3 weeks and
now he has the same initial symptoms.

eu

d. What is the commonest cause of his relapse?

----------------------------------------------------------------e. What is the likely cause of her MCV value?


------------------------------------------------------------------

[31]

www.Neurology4MRCP.com

Dr.Osama S.M Amin MRCPI MRCPS(Glasg)

MRCPI Part II Written Mock Test

Q4) A 34-year-old man presents with headache and impotence. These are his bloods:
Serum prolactin
Serum FSH
Serum LH

25 ng/ml (normal 2-15 ng/ml)


0.1 iu/L (normal 0.9-15 iu/L)
0.3 iu/L (normal 1.3-13 iu/L)

a. Name 2 other blood investigations?


--------------------------------------------------------------------------------------------------------b. What other investigation would you order?

P.
co

c. His serum TSH is normal. Give 2 explanations?

------------------------------------------------------

eu

ro

lo
gy

4M
R

-----------------------------------------------------------------------------------------------------------------------------------------------------------------------------------------------------------------------------------------

[32]

www.Neurology4MRCP.com

Dr.Osama S.M Amin MRCPI MRCPS(Glasg)

MRCPI Part II Written Mock Test

Q5) A 44-year-old alcoholic man is brought to the ER with confusion and generalized tonic
clonic fits for few hours. The ERs lab has performed some blood and urine tests and these are:
Serum sodium
Serum potassium
Serum chloride
Serum bicarbonate
Blood urea
Serum creatinine
Serum glucose
Urine examination
Plasma osmolality

137 meq/L
3.7 meq/L
102 meq/L
10 meq/L
56 mg/dl
1.6 mg/dl
90 mg/dl
RBCs 10 per HPF, full of oxalate crystals.
325 mosm/Kg

P.
co

-------------------------------------------------------------------------

a. Interpret his blood tests?

b. What test would you choose for definite diagnosis?

--------------------------------------------------------------------------

4M
R

c. Name 2 specific treatments for this mans illness?

--------------------------------------------------------------------------

eu

ro

lo
gy

---------------------------------------------------------------------------

[33]

www.Neurology4MRCP.com

Dr.Osama S.M Amin MRCPI MRCPS(Glasg)

MRCPI Part II Written Mock Test

Q6) A 32-year-old man presents with rapidly progressive flaccid areflexic paraparesis over the
past 8 days. This is his CSF profile:
CSF
Opening pressure
Cells
Protein
Sugar
Gram stain
ZN stain
Blood sugar
WBCs
Hemoglobin
ESR
Stool examination

P.
co

15 cm H2O
4 , all are mononuclear
200 mg/dl
78 mg/dl
negative
negative
100 mg/d
5300/ml3
13.5 g/dl
12 mm/hour
no blood, pus, or parasites

a. Enumerate 3 causes of shortness of breath in this man?

-----------------------------------------------------------------------

4M
R

---------------------------------------------------------------------------------------------------------------------------------------------

lo
gy

b. Name 3 bad prognostic factors?

-----------------------------------------------------------------------

ro

-----------------------------------------------------------------------

eu

-----------------------------------------------------------------------

[34]

www.Neurology4MRCP.com

Dr.Osama S.M Amin MRCPI MRCPS(Glasg)

MRCPI Part II Written Mock Test

Q7) A 9-year-old boy is referred to you with jaundice. His GP has ordered these investigations:
Total serum bilirubin 3.1 mg/dl
Indirect serum bilirubin 2.9 mg/dl
AST 12 iu/L
Alkaline phosphatase 50 iu/L
Serum albumin 4.0 g/L
a. Name 2 differential diagnoses?
---------------------------------------------------------------------------------------------------------------------------------------------

P.
co

-----------------------------------------------------------------------

b. What 4 investigations would you do to differentiate between them?

-----------------------------------------------------------------------

-----------------------------------------------------------------------

eu

ro

lo
gy

4M
R

-----------------------------------------------------------------------

[35]

www.Neurology4MRCP.com

Dr.Osama S.M Amin MRCPI MRCPS(Glasg)

MRCPI Part II Written Mock Test

Q8) A 34-year-old woman has low grade fever and arthralgia. These are her investigations:
Hemoglobin 8.0 g/dl
WBCs 2700/ml3.
Reticulocyte count 4%
ESR 950 mm/hour
AST 15 iu/L
Serum albumin 31 g/L
ECG wide-spread ST segment elevation
Urine red cell casts and protein ++
CRP negative
Blood culture negative

P.
co

-----------------------------------------------------------------------

a. What is the likely diagnosis this woman has?

b. Name 3 causes of shortness of breath in this woman?

-----------------------------------------------------------------------

4M
R

-----------------------------------------------------------------------

eu

ro

lo
gy

-----------------------------------------------------------------------

[36]

www.Neurology4MRCP.com

Dr.Osama S.M Amin MRCPI MRCPS(Glasg)

MRCPI Part II Written Mock Test

Q9)A 73-year-old immigrant man from India has persistent fever, headache, cough with whitish
scanty sputum, and weight loss for 2 months. His work up is shown:
Hemoglobin
9.0 g/dl
Blood film
normochromic RBCs
WBCs
6200/ml3
Blood urea
30 mg/dl
Serum potassium
3.1 meq/L
Alkaline phosphatase
70 iu/L
Serum albumin
3.0. g/L
ESR
90 mm/hour
Urine
negative for protein, blood, and bilirubin
Chest X ray irregular patchy pneumonic patch in the right lower lung field with pleural effusion.

P.
co

-----------------------------------------------------------------------

a. Name 3 investigation to be done next?

-----------------------------------------------------------------------

-----------------------------------------------------------------------

4M
R

The above investigations are pending and the patients headache is increasing with new
development of diplopia.
b. What 2 investigations would you do next?

lo
gy

---------------------------------------------------------------------------------------------------------------------------------------------

ro

c. What is the treatment?

eu

---------------------------------------------------------------------------------------------------------------

[37]

www.Neurology4MRCP.com

Dr.Osama S.M Amin MRCPI MRCPS(Glasg)

MRCPI Part II Written Mock Test

Q10) A 22-year-old man is referred by his GP for further evaluation of skin rash. His skin rash
non-itchy non-vesicular skin on extensor elbows. He was given a diagnosis of dermatitis
herpetiformis. These are the results of his investigations:
Hemoglobin 8.2 g/l
Serum albumin 29 g/L
Total serum bilirubin 0.9 mg/dl
AST 10 iu/L
Urine normal finding
a. Do you agree with the provisional diagnosis? Why?
-----------------------------------------------------------------------

P.
co

b. What would immune fluorescence study of skin biopsy in dermatitis herpetiformis show?
-----------------------------------------------------------------------

c. The skin biopsy result turned to be consistent with dermatitis herpetiformis. Explain the
non-itch and the non-vesicular nature of the rash?

eu

ro

lo
gy

4M
R

---------------------------------------------------------------------------------------------------------------------------------------------------------------------------------------------------------------------------------

[38]

www.Neurology4MRCP.com

Dr.Osama S.M Amin MRCPI MRCPS(Glasg)

MRCPI Part II Written Mock Test

Photographic Materials

eu

ro

lo
gy

4M
R

P.
co

Total 12 in Number

[39]

www.Neurology4MRCP.com

MRCPI Part II Written Mock Test

4M
R

P.
co

Dr.Osama S.M Amin MRCPI MRCPS(Glasg)

eu

ro

lo
gy

Figure 1

[40]

www.Neurology4MRCP.com

MRCPI Part II Written Mock Test

4M
R

P.
co

Dr.Osama S.M Amin MRCPI MRCPS(Glasg)

eu

ro

lo
gy

Figure 2

[41]

www.Neurology4MRCP.com

MRCPI Part II Written Mock Test

4M
R

P.
co

Dr.Osama S.M Amin MRCPI MRCPS(Glasg)

eu

ro

lo
gy

Figure 3

[42]

www.Neurology4MRCP.com

MRCPI Part II Written Mock Test

4M
R

P.
co

Dr.Osama S.M Amin MRCPI MRCPS(Glasg)

eu

ro

lo
gy

Figure 4

[43]

www.Neurology4MRCP.com

MRCPI Part II Written Mock Test

Figure 5

eu

ro

lo
gy

4M
R

P.
co

Dr.Osama S.M Amin MRCPI MRCPS(Glasg)

[44]

www.Neurology4MRCP.com

MRCPI Part II Written Mock Test

4M
R

P.
co

Dr.Osama S.M Amin MRCPI MRCPS(Glasg)

eu

ro

lo
gy

Figure 6

[45]

www.Neurology4MRCP.com

MRCPI Part II Written Mock Test

P.
co

Dr.Osama S.M Amin MRCPI MRCPS(Glasg)

eu

ro

lo
gy

4M
R

Figure 7

[46]

www.Neurology4MRCP.com

MRCPI Part II Written Mock Test

4M
R

P.
co

Dr.Osama S.M Amin MRCPI MRCPS(Glasg)

eu

ro

lo
gy

Figure 8

[47]

www.Neurology4MRCP.com

MRCPI Part II Written Mock Test

Figure 9

eu

ro

lo
gy

4M
R

P.
co

Dr.Osama S.M Amin MRCPI MRCPS(Glasg)

[48]

www.Neurology4MRCP.com

MRCPI Part II Written Mock Test

Figure 10

eu

ro

lo
gy

4M
R

P.
co

Dr.Osama S.M Amin MRCPI MRCPS(Glasg)

[49]

www.Neurology4MRCP.com

MRCPI Part II Written Mock Test

4M
R

P.
co

Dr.Osama S.M Amin MRCPI MRCPS(Glasg)

eu

ro

lo
gy

Figure 11

[50]

www.Neurology4MRCP.com

MRCPI Part II Written Mock Test

4M
R

P.
co

Dr.Osama S.M Amin MRCPI MRCPS(Glasg)

eu

ro

lo
gy

Figure 12

[51]

www.Neurology4MRCP.com

Dr.Osama S.M Amin MRCPI MRCPS(Glasg)

MRCPI Part II Written Mock Test

eu

ro

lo
gy

4M
R

P.
co

This page is left blank

[52]

www.Neurology4MRCP.com

Dr.Osama S.M Amin MRCPI MRCPS(Glasg)

MRCPI Part II Written Mock Test

Best of Many/Answers
Q1) b.
Objective: review the diagnostic approach of chronic stable angina.
A common theme in the MRCP examination is the diagnostic approach of ischemic heart disease.
This man has many risk factors for ischemic heart disease (IHD) and his chest pain sounds
ischemic. His resting 12-lead ECG was normal and repeating the test now would add nothing.
Can we proceed with formal exercise stress ECG testing?

The answer is no for 2 reasons:

4M
R

P.
co

1. The patient has high pre-test clinical probability for ischemic heart disease; therefore, a
positive treadmill exercise ECG testing will add nothing to the diagnosis (however, it
might have a prognostic significance). On the other hand, if the test turns negative, this is
very likely representing a false-negative test and will definitely affect the management
plan.
2. The patient has a right-sided artificial hip with osteoarthritis of the left knee; he can not
cope with the stress testing!
Therefore, option a can be eliminated.

lo
gy

Pharmacologic stress testing is used when formal exercise stress testing is needed but can not be
applied (for example, amputated leg, hip fractureetc); so, option c can be cancelled.

eu

ro

When the resting 12-lead ECG is abnormal (LVH, previous MI, medication effectsetc), and
stress ECG testing is indicated, it will definitely influence the interpretation of the stress testing;
myocardial perfusion imaging seems therefore reasonable. Our resting ECG is normal, besides,
there is no need for stress testing in this man; hence, option d is out of the menu.

When the overall clinical picture is highly suggestive of ischemic heart disease, proceeding to
coronary angiography is the right step. This would show the coronary anatomy and tree, number
of vessels involved, degree of stenosis, and the state of collateralization.
Notes about the application of exercise stress ECG testing:
a. When the clinical pre-test probability (for IHD) is low (e.g. a 30-year-old woman with
precordial pain and MVP), a positive test will likely represent a false positive; one the
other hand, a negative test will be an expected finding and will not influence the
management plan.
b. When the clinical pre-test probability is intermediate (e.g. a 50-year-old hypertensive
man with exertional chest pain), a positive test is likely to be a true positive, and a
negative test would call for coronary angiography.
c. For high clinical pre-test probability, see notes above.
[53]

www.Neurology4MRCP.com

Dr.Osama S.M Amin MRCPI MRCPS(Glasg)

MRCPI Part II Written Mock Test

In general, when you face a question about the diagnosis of IHD, note the following:
a.
b.
c.
d.

Age of the patient.


Risk factor for IHD.
Limitations/contraindication for formal stress testing.
Indications for pharmacologic stress testing/myocardial scintigraphy.

Q2) d.
Objective: differentiate between treatments of small cell versus non-small cell lung cancers.

Many MRCP candidates still have some sort of confusion in respect to the treatment of lung
cancers.

P.
co

Generally, divide lung cancers into 2 groups: small and non-small cell lung cancer.

ro

lo
gy

4M
R

Small cell lung cancer is staged into 2 stages according to the Veterans' Affairs Lung Study
Group staging system; limited and diffuse. The limited cancer is limited to one hemithorax and
is within the field (port) of radiotherapy (this roughly corresponds to stages I through IIIB); this
form can be treated with radiotherapy alone. The diffuse one has diffused outside one
hemithorax and is not within the radio-therapeutic field (port); this stage is targeted by the
combination of chemotherapy and radiotherapy. Our patient has anemia with leuko-erythroblastic
blood picture that might well represent bone marrow infiltration by the malignant cells; this is a
diffuse small cell lung cancer and option d is the treatment of choice. Note that the low serum
potassium might well represent SIADH; a common association, which is usually asymptomatic
and is only a biochemical diagnosis. Small cell lung cancer has no surgical treatment with the
intent to cure.

eu

Non-small cell lung cancer encompasses squamous cell carcinoma, adenocarcinoma, and large
cell cancer. This group can be staged (stage I-IV, or TNM) and each stage has a treatment
modality. Surgical treatment is used in localized and locally advanced tumors.

Q3) c.
Objective: review the management of diabetic complications.
Diabetes mellitus is disease that can have many complications which are responsible for most of
its morbidity and mortality.
This woman has long-standing poorly controlled type I diabetes, diabetic retinopathy, peripheral
neuropathy, and diabetic nephropathy; all of them need to be addressed and managed
appropriately. In general, this woman needs:
1. Optimization of her blood sugar control. Her insulin regimen needs to be re-assed and
changed.
[54]

www.Neurology4MRCP.com

Dr.Osama S.M Amin MRCPI MRCPS(Glasg)

MRCPI Part II Written Mock Test

P.
co

2. Control of blood pressure. Hypertension is uncommon in young patients with type I


diabetes (unlike patients with type II disease, in which it is very common), and its
presence should prompt the physician search for diabetic nephropathy. ACE inhibitors
are excellent options to tackle both hypertension and the kidney disease.
3. Diabetes mellitus is a coronary artery disease equivalent. Serum LDL must be maintained
below 100 mg/dl. This womans lipid profile is highly atherogenic and needs to be
addressed using a statin with dietary plans to bring down serum cholesterol and LDLcholesterol back to their target. Raised serum triglyceride is usually seen in poorly
controlled diabetes and usually becomes lower with proper glycemic control.
4. Symptomatic treatment of her peripheral neuropathy with protection of the feet from
trauma and infections. Gabapentine can be used to alleviate her numbness and
paresthesias.
5. Her fundi should be examined with pupillary dilatation annually.
6. Diabetic nephropathy is managed by proper control of blood sugar and blood pressure
and the use of ACE inhibitors.

4M
R

The cause of her renal disease can be clearly ascribed to diabetes and going to renal biopsy is not
indicated. If the clinical history or lab findings (e.g. hematuria, palpable purpuraetc) are
pointing to a non-diabetic kidney disease, then a kidney biopsy is done.

Q4) d.

lo
gy

Objective: beware of drug-drug interactions in various disease states.

eu

ro

This man was doing well after having his surgery and the sudden deterioration in the context of
flu and a new treatment with no new physical finding should call for measuring plasma level of
cyclosporine. His family history is irrelevant to his new presentation and is just a distracter! His
past medical and surgical histories have nothing to do with his new presentation as well.
Cyclosporine is metabolized by the hepatic cytochrome P450 A3 system which is affected by
many medications and drugs. Clarithromycin, erythromycin, diltiazem, fluconazole, lanzoprazole,
colchicine, amiodarone, and grapefruit juice inhibit this enzymatic system and may prominently
elevate the serum level of cyclosporine, and therefore, clinical toxicity can ensue rapidly.
Rifampin, phenytoin, phenobarbitone, and carbamazepine have the opposite effect and may result
in immune suppression failure (and graft rejection for example in this man). Neurological
complications are usually reversible upon lowering the daily drug dosage (or stopping it briefly if
the condition permits) or changing the intravenous form to oral formulations.

Q5) b.
Objective: review the causes of iron deficiency anemia and their diagnosis.
This mans clinical picture of hypochromic microcytic anemia with dyspeptic symptoms should
always call for searching a GIT source of blood loss. His family history has 2 GIT diseases; one
[55]

www.Neurology4MRCP.com

Dr.Osama S.M Amin MRCPI MRCPS(Glasg)

MRCPI Part II Written Mock Test

of them is a GIT malignancy. Although he has no bowel habit change, his GP was right by
ordering colonoscopy, the result of which is normal. A good step to do now is to proceed with
upper GIT endoscopy. Fecal occult blood testing is neither sensitive nor specific for GIT
malignancies, and the test can be truly negative while the patient has a genuine GIT blood loss.
Repeating colonoscopy or doing sigmoidoscopy will be unreasonable. Bone marrow study would
confirm the iron deficiency anemia but will not uncover its cause.

Q6) e.
Objective: be familiar with the management of multiple sclerosis.

lo
gy

4M
R

P.
co

This woman presents for the first time with an attack of retrobulbar optic neuritis. Her history did
not show any neurological complaints in other systems (paresthesia, weakness, sphincters,
unstable stanceetc.) and her investigations are normal apart from her brain MRI findings of
periventricular plaques. We can label her as having a clinically isolated syndrome (CIS)
suggestive of multiple sclerosis. About 80% of CIS patients with abnormal brain MRI in
general will go to develop the fully fledged multiple sclerosis after some time. Around 15-75% of
those who present with optic neuritis will be diagnosed eventually with multiple sclerosis. There
is an accumulating evidence that starting beta interferon (disease-modifying) therapy early in the
course of these CISs can improve multiple sclerosis outcome in the long-term (which is definitely
better that the wait and see policy).

Q7) d.

ro

Objective: review the ocular complications of rheumatoid arthritis and their management.

eu

She has developed secondary Sjgrens syndrome with dry eyes (and possibly dry mouth). The
best treatment of the ocular dryness is artificial eye drops (like methylcellulose or polyvinyl
alcohol) and artificial saliva sprays can be used for the oral dryness; note that the treatment of
Sjgrens syndrome is largely symptomatic. Treating the underlying disease in secondary cases
definitely needs attention but per se will not alleviate the sicca symptoms. This lady absolutely
needs modification of her current therapy to bring the disease to remission and control, but the
question has addressed the treatment of the ocular complaints only.

Q8) c.
Objective: review the treatment and lab follow-up of hypothyroidism.
Although the patient is symptomatically well and her daily dosage of L-thyroxin is stable since 8
months, her lab results are disappointing. The objective of treating primary hypothyroidism is to
bring the serum TSH within its normal reference range; this patients serum TSH is far from this.
Simply, increasing the daily dose to 100 g per day with serial follow-ups is needed to see what
[56]

www.Neurology4MRCP.com

Dr.Osama S.M Amin MRCPI MRCPS(Glasg)

MRCPI Part II Written Mock Test

daily dose keeps this serum TSH within its reference range (keeping her with the same daily dose
is not acceptable). The patient is not over treated (no suppressed TSH) so as to decrease the daily
L-thyroxin dose. The hypothyroidism is not secondary (with low ACTH); therefore, no need for
glucocorticoid replacement therapy. For the same reason, growth hormone replacement has no
place.
Note that when starting treatment for primary hypothyroidism, serum T4 normalizes first and the
serum TSH lags behind for a few weeks and even months; this should be kept in mind in the
follow-up visits.

Q9) c.

P.
co

Objective: review interpretation of liver function tests and their correlation with the clinical
picture.

ro

lo
gy

4M
R

This chronic schizophrenic man has developed jaundice with itching and a lab evidence of
cholestatic hepatitis; option c would fit the picture. Note that the generalized
hyperpigmentation is due to chlorpromazine and is not ascribed to primary biliary cirrhosis (this
patient has no clubbing and the history is short, besides, the question did nor address a positive
anti-mitochondrial antibodies). Non-alcoholic hepatic macrosteatosis typically has normal or
slightly raised serum transaminases (usually ALT); jaundice and cholestasis are not features of
this disease. The drinking history does not refer to a possible development of alcoholic hepatitis.
The presence of positive IgG anti-HBs antibodies indicates past vaccination (although the patient
did not tell us about this); positive serum testing for HBs antigen with IgG anti-HBc antibodies is
seen in chronic hepatitis B infection.

eu

Q10) a.

Objective: review acute HIV seroconversion illness.


The patient displays a high risk behavior for HIV infection; although his sex with men is
protected, he is still at risk of acquiring the infection. Acute HIV seroconversion illness usually
occurs 2-4 weeks post -exposure; up to several months may elapse before the appearance of this
syndrome in some patients. The HIV viral load in plasma is typically high and the viremia makes
the patient highly infectious to others. Oral ulcerations are seen in 20% of cases, and the
meningitic process is due to the virus itself. Factors that may indicate rapid progression to AIDs
are oral thrush, low CD4+ count, prolonged illness, and presence of opportunistic infections at the
time of the diagnosis. Epstein-Barr viral mononucleosis ranks the first on the list of the
differential diagnoses; heterophil antibodies are uncommonly positive in HIV seroconversion
illness and will pose a diagnostic dilemma.

[57]

www.Neurology4MRCP.com

Dr.Osama S.M Amin MRCPI MRCPS(Glasg)

MRCPI Part II Written Mock Test

Q11) d.
Objective: review the management of asthma according to its severity.

P.
co

This man has a sudden deterioration in his asthma control after an apparently mild upper
respiratory tract infection. The question did not mention the PEFR recordings and did not include
any clinical life-threatening parameter. However, his PCO2 is within its normal reference range;
However, CO2 washout is an expected finding with hypocarbia in this. This indicates that the
patient has impending respiratory failure and going to mechanical ventilation is the correct action
to save the patients life. Montelukast is used in step 4 of asthma treatment in the long-term and
has no place in acute severe attacks. Patients who are still prominently symptomatic while being
managed for their acute asthma can receive intravenous magnesium. Oral (or intravenous)
glucocorticoids are used in acute asthma attacks but they should not be used alone in the
treatment of this patients condition as a life-saving measure. The development of uni- or bilateral
pneumothoraces is a well-recognized acute asthma complication that can destroy the management
plan; should always be suspected in acute asthma attacks with lateralized chest pain (uni-or
bilateral). This patients X-ray findings excluded this possibility.

4M
R

Q12) c.

Objective: differentiate between the treatments of STEMI vs NSTEMI.

eu

ro

lo
gy

This patient has developed non-ST segment elevation myocardial infarction (NSTEMI) with a
TIMI score of 5 out of 7; this would put him in the high risk category. NSTEMI patients with
intermediate (TIMI score of 3-4) or high (TIMI score of 5-7) risk patients benefit from early
coronary intervention once the patient is stabilized on optimal medical therapy. High risk patients
should also receive clopidogrel and GPIIb/IIIa inhibitors (like abciximab). Direct thrombin
inhibitors (e.g. lepirudin) are suitable alternatives to unfractionated heparin when the latter is
contraindicated (as in those with a history of heparin-induced thrombocytopenia). Thrombolytic
therapy has no place in NSTEMI. ACE inhibitors can be used when indicated (e.g. to control
blood pressure, tackle congestive heart failure...etc) but by their own they are not part of the
treatment of NSTEMI per se. Stress ECG testing can add prognostic significance, but is usually
deferred until the patient stabilizes after at least 1 month.
Please review TIMI score.

Q13) e.
Objective: review the causes of acute interstitial nephritis.
This mans illness started with bacterial pharyngitis that is responsive oral ampicilline therapy.
However, later on, he developed fever, arthralgia, and skin rash with non-oliguric acute renal
impairment, eosinophilia and eosinophiluria. Drug-induced acute allergic interstitial nephritis
would fit this clinical scenario. His history of gonorrhea is unrelated by any way to his current
[58]

www.Neurology4MRCP.com

Dr.Osama S.M Amin MRCPI MRCPS(Glasg)

MRCPI Part II Written Mock Test

presentation and option b is a distracter. Option a, as in post-streptococcal


glomerulonephritis, presents with rapidly progressive renal failure and diminished urine output,
edema, hypertension, and active urinary sediment. Option c is a possibility with the
development of skin rash after starting ampicilline. However, the virus rarely tackles the kidneys
in such degree and does not produce eosinophilia. Hanta virus can attack the lung and kidneys but
it has a very different clinical picture (which may include a hemorrhagic fever presentation).

Q14) c.
Objective: review the mode of presentation of Addisons disease and its diagnosis.

lo
gy

4M
R

P.
co

This mans old chest tuberculosis might well have had involved the adrenals to produce
Addisons disease which is responsible for this mans current non-specific presentation.
Addisons can mildly elevate blood urea and may produce hyperkalemia, hypercalcemia,
hypoglycemia, anemia, and eosinopenia. Early morning serum cortisol is usually high in normal
conditions; low serum levels with high serum ACTH should call for further evaluation of
Addisons. Short ACTH stimulation test should be performed virtually in all patients in whom the
diagnosis is being considered. Growth hormone deficient patients are usually plumb, and may
have fatigue, but the deficient hormone does not produce tanning or the blood results mentioned
in the question. Renal failure patients may present solely with malaise, but there is earthy-colored
face rather than generalized tanning, and the degree of renal impairment is much deranged.
Options a and b are used in the overall management plan of Addisons, but they are not
diagnostic by their own.

ro

Q15) c.

eu

Objective: review chronic lymphocytic leukemia (staging, management, complications).

About 25% of CLL cases are diagnosed incidentally by performing blood counts for some reason
or another. Lymphocytosis with small mature-looking lymphocytes (and smudge cells) is the first
clue in such cases. This man has no enlarged lymph nodes or organomegaly. This would
categorize him as having stage 0 (according to Rais system) which confers a median survival of
150 months; so, observation is all that required. Patients with repeated major infections
(septicemia, severe pneumoniaetc) should receive monthly IVIG infusions. Short doubling time
portends bad prognosis. Around 10% of CLL patients will have Richters transformation, and
another 10% will have prolymphocytoid transformation. Transformation to acute leukemias is
unusual. However, CLL patients have greater than average risk of developing solid malignancy
(usually of the lung, GIT, and skin).

[59]

www.Neurology4MRCP.com

Dr.Osama S.M Amin MRCPI MRCPS(Glasg)

MRCPI Part II Written Mock Test

Q16) e.
Objective: review the characteristics of seronegative spondarthritides.

P.
co

Although the question did not mention any skin lesions or nail manifestations, the combination of
spinal complaints, distal interphalangeal joint swelling (and hand soreness) with raised ERS and a
positive family history of ankylosing spondylitis should alert you about the possibility of psoriatic
arthritis. Note that familial clustering in seronegative spondarthritides is common and is not
necessarily of the same disease. Many features overlap (e.g. skin, eye, enthesopathy) and some
cases may not have a clear cut diagnosis. Rheumatoid factor and ANA are negative. The first line
medications are NSAIDS; if no response or the disease is severe and polyarticular, disease
modifying antirheumatic drugs should be started. Methotrextae, cyclosporine, and retinoic acid
derivatives have been shown to improve the skin and articular manifestations; however, none of
them has been shown to retard or at least prevent joint damage! Anti-TNF alpha preparations
(like inlfiximab) are showing promising results but more studies are needed to confirm this
observation, and they can be given once DMARDs have proven ineffective after at least a 6month trial.

4M
R

Q17) c.

Objective: review the different modalities used in the treatment of chronic inflammatory
demyelinating polyradiculopathy (CIPD).

eu

ro

lo
gy

Oral prednisolone, plasma exchange, and IVIG are equally effective at treating CIDP. However,
each mode of treatment has pros and cons. His diabetes is unstable and he does not want insulin
for optimal control, and besides, he has osteoporosis; these would make prednisolone therapy out
of question. Patients with cardiovascular instability, as our patient with congestive heart failure,
are unable to tolerate the fluid shifts incurred by plasma exchange; therefore, this modality can be
crossed out. He does not have motor neuron disease (there are prominent sensory signs and
symptoms); hence, riluzole has no place. The overall clinical picture is far away from multiple
sclerosis and option e can be cancelled. IVIG is the best treatment option in this man with comorbidities. Patients usually start to show improvement after 5 days and this improvement is
usually maintained for 2-4 weeks; therefore, repeated administration at 4-week intervals sounds
reasonable. IVIG uncommonly can result in headache, renal shutdown, aseptic meningitis, and
Coombs positive hemolytic anemia. Note that this form of therapy may affect some lab tests in
some way or another transiently:
1.
2.
3.
4.

May raise plasma viscosity.


May reduce the ESR.
May decrease the anion gap (because it has many positively charged components).
Antibody titers (ANA, rheumatoid factor, viral antibodiesetc) can not be relied upon
shortly after the infusion (a positive test may simply be due to the donors antibodies).
5. Dilutional hyponatremia and pseudnatremia.
6. Neutropenia.
[60]

www.Neurology4MRCP.com

Dr.Osama S.M Amin MRCPI MRCPS(Glasg)

MRCPI Part II Written Mock Test

Because of its relative safety, ease of administration, as well as being convenient to many
patients, the American Academy of Neurology guidelines recommends this form of therapy as the
first line treatment in CIDP in general.

Q18) e.
Objective: review the follow-up plan of peptic ulcers.

4M
R

P.
co

The 2 commonest causes of medical treatment failure of peptic ulcers are medications noncompliance and H.pylori resistance to antibiotics. Given the full compliance of the patient with
his treatment regimen, the second option seems likely. To be sure of non-eradication of H.pylori
in this duodenal ulcer, simply do urea breath test, which if positive, this means that the organisms
is still there and active and the symptoms can be ascribed to treatment failure (whatever the
reason was). Although the endoscopist did not take a biopsy from his duodenal ulcer, repeating
the endoscopy is needed only in gastric ulcers (even if they are benign-looking) to be sure of
complete healing. Barium meal lacks sensitivity and specificity. The organism might well have
resistance to anti-H.pylori treatment and continuing the same regimen would sound unwise.
Simply, in this man, do urea breath test, which if positive, give a different anti-H.pylori
eradication therapy. Barium follow through have no place in the diagnosis or follow up of
duodenal ulcers.

lo
gy

Q19) b.

ro

Objective: differentiate between dengue fever from dengue hemorrhagic fever and know their
differential diagnoses.

eu

This patient travelled for the first time to an area thas is endemic in dengue. It is the commonest
mosquito-born infection world-wide (over 100 million dengue viral infections occur annually
throughout the world). The patient did not recall a mosquito bite; however, the bite might be
minor and easily forgotten by the patient; absence of such bite does not refute the diagnosis. After
his exposure to the virus (the incubation period is 3-14 days; dengue can be excluded when the
traveler develops fever after 2 weeks of his return), the patient developed the typical features of
classical dengue fever. About 55% of infections after the first exposure are either asymptomatic
or minimally symptomatic. Dengue hemorrhagic fever (note the absence of hemorrhagic
manifestations in this man) needs a past-exposure to the wild virus to occur (our patient went to
an endemic area for the first time in his life). To date, there is no licensed vaccine to protect
against dengue. In the MRCP examinations, note that dengue hemorrhagic fever in people from
non-endemic areas mainly occurs in those with frequent international travel or in immigrants
from high endemic areas upon their return to their country of origin. Options a and c are not
seen in Thailand. Option e can be safely excluded by the absence of sexual history or other high
risk behaviors.

[61]

www.Neurology4MRCP.com

Dr.Osama S.M Amin MRCPI MRCPS(Glasg)

MRCPI Part II Written Mock Test

Q20) c.
Objective: review the role of biventricular pacing in congestive heart failure.
Biventricular pacing (resynchronization therapy) has been shown by many studies to improve the
cardiac pump and contractile function and can reserve ventricular remodeling; ventricular
dyssynchrony exacerbates LV dysfunction. This from of therapy is used in highly selected patient
with:
1. Advanced heart failure (usually NYHA functional class III or IV).
2. Severe LV dysfunction (ejection fraction usually lower than 35%).
3. Intraventricular conduction delay (QRS complex duration > 120 msec).

lo
gy

4M
R

P.
co

This patients medical treatment (an ACE inhibitor, a beta blocker, an aldosterone antagonist, and
a loop diuretic) has resulted in some degree of symptomatic and clinical improvement; however,
she is a good candidate for resynchronization therapy for a better response to occur. Digoxin may
reduce hospitalization rate and may produce some symptomatic improvement even in those with
sinus rhythm but has no survival benefit; however, it is unlikely that it will produce substantial
improvement in this woman. The question did not address any form of cardiac arrhythmia.
Cardiac transplantation is an option in refractory heart failure (which is not the case here).
Although her JVP is mildly elevated, her chest has no basal crackles and her legs have trace leg
edema; her volume status is reasonably controlled and there is no need to increase the diuretic
doses.

Q21) d.

ro

Objective: review the management of nephrotic syndrome in children.

eu

This boy has obviously failed his medical treatment and iatrogenic Cushings syndrome has
ensued. His urine is grossly proteinuric and he is clinically edematous. The boy was diagnosed
with nephrotic syndrome by the combination of his clinical features and urinary and bloods
findings and was given a treatment for a presumed minimal changed disease. Pediatric
nephrologists avoid doing renal biopsy because minimal change disease is the commonest cause
of nephrotic syndrome in children, and they do renal biopsy whenever the clinical picture is
atypical (hypertension at the time of diagnosis, hematuria, impaired renal functionetc) or the
syndrome is unresponsive to steroids (as in our boy). Note that this kids hypertension is due to
his high dose prednisolone.
Generally, 90% of children (with suspected minimal change disease, as in our patient) will
respond to steroids within 4 weeks. The rest will usually respond after a further 2-4 weeks period.
Those who dont respond after this period may try methylprednisolone pulses and should be
reviewed after 1 week; if urine is still grossly proteinuric, the patient is labeled as having steroidresistant nephroic syndrome and renal biopsy should be done.

[62]

www.Neurology4MRCP.com

Dr.Osama S.M Amin MRCPI MRCPS(Glasg)

MRCPI Part II Written Mock Test

Adult nephrologists prefer to do renal biopsy at the time of presentation because they believe that
knowing the etiology of nephrotic-range prtienuruia would help decide the best treatment plan by
finding an unexpected diagnosis through histological examination.

Q22) c.
Objective: review the different treatment modalities of pneumothorax.

4M
R

P.
co

The patient has developed primary spontaneous non-traumatic left-sided pneumothorax that
gradually has become under tension. Pneumothorax with cardio-respiratory comprise is
considered to be under tension regardless of it size; note the patients hypotension and severe
dyspnea. Chest tube should be inserted, and when no improvement is seen (no resolution of the
pneumothorax), applying suction on the tube is recommended. Please review the full management
of pneumothorax as it is commonly seen in the examination. There are no cardiac abnormalities
and option e can be cancelled. The pneuomothorax is on the left ad there is no indicator of
bilateral occurrence so as to insert another chest tube in the right side. The pneumothorax is under
tension and the patient is unstable and need immediate relief by chest tube insertion; there is no
place for chest aspiration here.

Q23) d.

lo
gy

Objective: review the diagnostic approach of acromegaly.

eu

ro

Hyperglycemia, bilateral carpal tunnel syndrome, and hypertension all can occur in acromegaly.
The presence of the above features in someone with large head has alerted the GP about the
possibility of acromegaly and he has already arranged a diagnostic plan for it. Most diagnostic
guidelines start with measurement of the IGF-I; if normal, acromegaly can be excluded and no
need for further testing. However, if IGF-I is elevated, one can confidently proceed with
measurement of serum growth home during oral glucose tolerance test. If the latter test shows
inappropriate suppression (adequate suppression is inconsistent with acromegaly), brain MRI is
the next step to image the pituitary. If the pituitary is normal, chest and abdomen CT scanning is
done with measurement of the serum GHRH level.
This man demonstrated low normal level of serum IGF-I; this means that he has no acromegaly.

Q24) e.
Objective: beware when and how to treat Pagets disease of the bone.
The usual indications of treatment with specific anti-Pagetic medications (bisphosphonates and
calcitonin) in symptomatic patients are: neurological complications (nerve compression, basilar
invaginationetc), headache due to skull involvement, and prominent pain syndromes not
[63]

www.Neurology4MRCP.com

Dr.Osama S.M Amin MRCPI MRCPS(Glasg)

MRCPI Part II Written Mock Test

responsive to simple analgesics, as well as immobilization hypercalcemia (as in our patient). Note
that bowed legs will not correct themselves and deafness is unlikely to improve, while pain due to
secondary osteoarthritis may or may not improve upon receiving such therapy.
The indications of treatment of asymptomatic patients are less clear. However, treatment can be
started when any one of the following is present:
1. Extensive skull involvement.
2. Disease involving the tibia or femur in which progression is likely.
3. Serum alkaline phosphates that is at least 4 times its upper normal limit (i.e. indicating
moderately active disease) in patients with bone involvement at sites liable for severe
complications (like near joints, weight-bearing joint, vertebral bodyetc).

P.
co

Our patient is minimally symptomatic but has immobilization hypercalcemia. He should receive
medical therapy.

Q25) c.

Objective: differentiate between hepatitis B and its variants.

Q26) d.

eu

ro

lo
gy

4M
R

The patient is at risk of hepatitis B, hepatitis C, and HIV infections because of homosexuality and
intravenous drug abuse. The latter 2 have been excluded by appropriate testing. There are no
clues to the presence of hepatocellular carcinoma (liver mass, prominent elevation in alpha
fetoproteinetc) or cirrhosis (compensated or uncompensated). The question addressed hepatitis
B status of the patient. HBs antigen marks infection, and the elevated plasma HBV DNA level
indicates high infectivity of the virus, but the negative HBe antigen (a marker of infectivity)
would normally cancel this possibility. The combination of chronic hepatitis B infection with
elevated plasma viral DNA, negative HBe antigen, and persistent (or intermittent) elevation in
liver aminotransferases (that can not be ascribed to other causes) reflects an infection with a HBV
variant (usually a pre-core mutant or core-promoter variants).

Objective: review the different options for treating essential tremor (ET).
The overall clinical scenario points towards essential tremor. The American Academy of
Neurology recommends either propranolol or primidone as a first line treatment. This patients
asthma would cancel propranolol (however, low doses of a selective beta blocker, like atenolol,
have been advised in such cases). Botulinum toxin injection is an excellent option in medically
refractory limb, head, and voice tremors. Alprazolam (or clonazepam) are second line agents
given their physical dependence potential as well as problems with drug withdrawal; these agents
are actually widely used because of the wrong belief that hand shaking is due to anxiety. Alcohol
has long been shown that it relieves essential tremor, especially ET-related gait ataxia, and
drinking small amounts before meals or at social events is actually practiced by many patients
[64]

www.Neurology4MRCP.com

Dr.Osama S.M Amin MRCPI MRCPS(Glasg)

MRCPI Part II Written Mock Test

who seek temporary relief ; however, because of the possibility of misuse and abuse, and the
problem of withdrawal, this form of treatment can not be advised to all patients. Other agents that
can be used are gabapentine and topiramate; note that the combination of propranolol and
primidone is better than either alone.

Q27) b.
Objective: review the management of atopic dermatitis.

4M
R

P.
co

This woman displays features of atopic dermatitis. Personal or family history of atopy is
common. The skin loses its barrier function and becomes vulnerable to a variety of bacterial,
viral, and fungal infections, which will exacerbate the skin lesions; therefore, skin infection is
both a trigger and a complication of atopic eczema. Atopic eczema has no correlation with
pemphigus. The cornerstone in the treatment (besides avoiding skin irritants and proper treatment
of superadded infections) is the combination of topical emollients and topical steroids, with
topical calcineurin inhibitors added (tacrolimus, pimecrolimus) when the rash involves the face or
skin folds. Systemic steroids are used when topical measures fail.

Q28) e.

Objective: review the treatment of tricyclic anti-depressants (TCAs) intoxication.

eu

ro

lo
gy

Although selective serotonin reuptake inhibitors have become the first line agents for the
treatment of depressive disorders, trcicyclics are still used for depressed patients a well as in
chronic pain syndromes, obsessive-compulsive disorders, and panic disorders, and clinical
intoxication (whether intentional or accidental) is still seen. TCAs intoxication usually produces
sinus tachycardia and hypotension; VT and VF are seen in only 4% of cases. Actually, refractory
hypotension is responsible for the majority of TCA-related mortality. Bicarbonate infusion
usually improves hypotension (and shortens the prolonged QRS duration) and crystalloids have
an additive effect. However, when the blood pressure fails to rise, intravenous vasopressors
should be administered (like noradrenalin in this patient). Note that sodium channel blockers (like
phenytoin) are ineffective in the treatment of toxin-medicated seizures (benzodiazepines are the
agents of choice to increase the CNS GABAergic transmission). TCAs are not dialyzable, as they
tightly protein-bound. Although severe and advanced cases can have apnea, the question did not
address any respiratory compromise and option a is not applicable. Her ECG did not show
polymorphic VT and option d can not be given as a prophylaxis. Epicac is no longer
recommended for upper GIT decontamination.

[65]

www.Neurology4MRCP.com

Dr.Osama S.M Amin MRCPI MRCPS(Glasg)

MRCPI Part II Written Mock Test

Q29) d, h.
Objective: review the management of metastatic breast cancer.
Around 10% of women with breast cancer have metastatic disease at the time of the diagnosis;
the majority develops such dissemination after being treated for localized or locally advanced
breast cancer. This woman obviously has wide-spread disease and she requests treatment.
Treatment should be frankly discussed with all such patients saying that cure is very unlikely,
complete remission from chemotherapy is uncommon, and that treatment carries considerable
toxicity for getting a marginal survival benefit. However, those who respond are usually young,
have limited disease (oligometastatic disease), and with excellent pre-treatment functional status.

lo
gy

4M
R

P.
co

Our patients tumor is estrogen negative; therefore, endocrinal manipulation (tamoxifen,


anastrazoleetc) is virtually useless. Trastuzumab (Herceptin) has been shown to be effective
in tumors which express high levels of HER2; those with very low (as in our patient) or no
HER2, dont benefit from this mode of therapy. Localized radiotherapy can be delivered for
locally painful skeletal areas, but irradiation of lung secondaries is not applicable. The question
did not give a clue to intracranial secondaries; so, option a is cancelled. Surgical resection of
localized breast secondary may be applied in selected cases, but our patient has no such a thing;
radical mastectomy is used in stage I or II disease. She has bone marrow involvement as evident
by the blood film result; this may respond to chemotherapy (no place for bone marrow
transplantation). Bisphosphonates have been shown to reduce skeletal events in metastatic (lytic,
and probably blastic as well) breast cancer.

Q30) a, j.

ro

Objective: review the diagnosis of fibromyalgia.

eu

The combination of clinical features, normal lab investigation with multiple tender points can
secure the diagnosis of fibromyalgia. The blood tests should be limited to routine ones as well as
TSH and muscle enzymes, which all should be normal. More extensive testing is not indicated
and may even give confusing results. e.g. ANA can be found in normal healthy individuals, and
unless the clinical picture is suggestive of SLE, ordering this test will be counterproductive.
Option i can be seen in irritable bowel syndrome (which this woman also manifests as part of
her fibromyalgia). Raised ESR and thrombocytopenia are never normal accompaniment of
fibromyalgia and their presence should cast a strong doubt on the diagnosis. The list of
differential diagnoses of fibromyalgia is long and includes hypothyroidism, depression,
polymyalgia rheumatica, connective tissue disorders (especially rheumatoid arthritis, SLE, and
Sjrgrens syndrome), inflammatory myositis and metabolic myopathy. Note that fibromyalgia is
a chronic pain disorder that is difficult to treat. The cornerstone of the treatment is reassurance
and careful explanation of the disease combined with low dose TCAs at night, graded aerobic
exercises, and cognitive behavioral therap.

[66]

www.Neurology4MRCP.com

Dr.Osama S.M Amin MRCPI MRCPS(Glasg)

MRCPI Part II Written Mock Test

Case Histories/Answers
Q1)
a. Ebsteins anomaly with ostium primum atrial septal defect (and Wolf-Parkinsons White
syndrome).
b. Zero percent.
c. Paradoxical embolism.

P.
co

The auscultatory findings are pointing towards tricuspid regurgitation. She has prolonged PR
interval, left axis deviation, and partial RBBB. Her mother might well have ingested lithium as
part of her bipolar treatment. The above combination would fit Ebsteins anomaly. Note that 52%
of those patients have single accessory pathway while 29% have multiple pathways resulting in
WPW syndrome; about 18% have arrhythmia unrelated to accessory pathway presence. The
disease is sporadic with congenital malformation of the tricuspid valve and right ventricle. Some
cases are associated with maternal lithium ingestion; the disease is not inherited in a particular
genetic pattern. Right to left shunt through patent foramen ovale or ASD is common.

4M
R

In Ebstiens anomaly, the tricuspid valve has the following characteristics:

Q2)

eu

ro

lo
gy

1. The anterior leaflet is usually the largest one and is usually attached normally to the AV
ring.
2. The posterior and septal leaflets are vestigial (and sometimes may be totally absent);
when present, they are usually displaced downward far away from the AV ring into the
cavity of the right ventricle (or even the right ventricular apex).
Overall , the tricuspid valve is sail-like and incompetent (and might even be stenotic).

a. Join pain (or back pain, or heel pain).


b. Reiters syndrome.
c. Keratoderma blenorrhagica, circinate balanitis.
The combination of urethritis, bilateral conjunctivitis, and arthritis suggests Reiters.
After the initial attack of reactive arthritis, patients in general have 1 out of 4 of the following
possibilities:
1. The arthritis subsides completely and never recurs but may last for up to 6 months. This
is seen in 35% of cases.
2. The disease enters a remission phase, but occurs intermittently at irregular intervals.
These relapses may consist only of arthritis, enthesopathy, or an extra-articular disease
(like iritis). Like the first group, this category occurs in 35% of cases.
[67]

www.Neurology4MRCP.com

Dr.Osama S.M Amin MRCPI MRCPS(Glasg)

MRCPI Part II Written Mock Test

3. The initial arthritis never subsides but continues with a chronic fluctuating course. This
targets 25% of cases.
4. Up 5% of patients will continue to have severe inflammatory disease from the outset
which may continue for years and may result in severe destructive peripheral arthritis or
diffuse spinal involvement in ankylosing spondylitis-like picture.

Q3)

a. Bilateral chronic subdural hematomas.


Massive right hemispheric ischemic stroke.
b. Subdural hematomas.
c. Oral anticoagulation.
Atrial fibrillation.

P.
co

This man presents with massive ischemic stroke involving the main stem of the right middle
cerebral artery. This artery is usually blocked by an embolus (?atrial fibrillation). The man has
bilateral subdural hematomas, which are hypodenser than the brain parenchyma; their age is
greater than 21days (?anticoagulant use).

ro

lo
gy

4M
R

Subdural hematoma is bilateral in 1/6th of cases and the older the patient the less the likely history
of head trauma. The characteristic fluctuating consciousness is uncommon in clinical practice.
Suacute bilateral hematomas can be totally isodense with the brain parenchyma and the brain CT
scan report of these patients is normal; completely normal or hyper-normal brain CT scan in
elderly people should always prompt the physician search for bilateral subacute subdural
hematomas.

eu

Q4)

a. Right basal pneumonia.


b. Complete blood count.
Blood culture (sputum culture, sputum smears).
Abdominal ultrasound.
c. Lung abscess.
Empyema thoracis.
Basal pneumonia can present with abdominal symptoms. Abdominal pathologies one the other
hand can result in chest symptoms (for example, subphrenic abscess can result in reactionary
pleural effusion). Persistence or recurrence of fever should always prompt the physician search
for empyema (or complicated pleural effusion), lung abscess, metastatic abscesses, canulaassociated thrombophlebitis, and drug fever.
The incidence of parapneumonic effusions is increasing and S.pneumoniae (especially antibioticresistant strains) ranks the commonest cause among the bacterial causes of pleural effusion. Every
[68]

www.Neurology4MRCP.com

Dr.Osama S.M Amin MRCPI MRCPS(Glasg)

MRCPI Part II Written Mock Test

patient with parapneumonic effusion deserves pleural aspiration and analysis when the amount is
more than trivial. Indicators of complicated effusion are pleural fluid PH less than 7.0, glucose
<40 mg/dl, or LDH >1000 iu/L. Frank pus aspiration is gross empyema.

Q5)
a. Recurrent hyperthyroidism.
b. Subtotal thyroidectomy.
c. Radioiodine (or carbimazole or propylthiouracil)

P.
co

Notes about pregnancy and radioiodine therapy:

Note the collar scar of previous surgery. Surgical ablation of the thyroid is usually chosen as a
first line treatment in those with severe hyperthyroidism or in those with large goiter. Recurrence
of the disease would call for another form for treatment, preferably using radioiodine.

lo
gy

4M
R

Radioiodine is absolutely contraindicated in pregnancy. Some physicians start this therapy only
after doing pregnancy test for sexually active females. All women should advised to postpone
pregnancy for about 4-6 months; when pregnancy occurs unintentionally during this period or this
mode of treatment is given inadvertently to pregnant women, the fetus thyroid function can be
assessed in utero by umbilical cord sampling after the 20th weeks of gestation; or else, appropriate
action should be taken immediately after birth. Those fetuses could be normothyroid,
hypothyroid, or even hyperthyroid.

Q6)

eu

ro

a. Multiple endocrine neopalsia type I


b. Hypercalcemia.
Cranial diabetes insipidus.
c. Yes. It is an autosomal dominant disease.
The combination of hypoglycemia (insulinoma), polyuria (hypercalcemia), and large hands
(acromegaly) can fit MEN I. The tumor could encroach upon the hypothalamus producing cranial
diabetes insipidus. Note that up to 10% of insulinomas have MEN I. Primary
hyperparathyroidism is seen in at least 90% of MEN I patients and symptomatic or asymptomatic
diseases are treated in the same way as sporadic cases.

[69]

www.Neurology4MRCP.com

Dr.Osama S.M Amin MRCPI MRCPS(Glasg)

MRCPI Part II Written Mock Test

Q7)
a. Lyme disease.
b. 12-lead resting ECG.
Lyme serology.
CSF analysis.
(brain CT scan/MRI with contrast)
c. Ceftriaxone.
d. Bebesiosis.
Human granulocytic ehrlichiosis

Note that this is a late disseminated disease, and treatment with ceftriaxone is recommended. For
early disease, doxycyclin, amoxicilline, or cefurixime are recommended. If the question had not
mentioned the characteristic skin lesion, sarcoidosis would be a differential diagnosis.

lo
gy

4M
R

P.
co

Serological diagnosis of Lyme starts with ELISA, which if positive or intermediate, proceeding to
Western blot is recommended. False positive ELISA (i.e. positive ELISA but negative Western
blot) is common and may occur with other Borrelial infections (like relapsing fever), spirochetal
infections (syphilis, yaws, leptospirosisetc), many viral infections, and autoimmune diseases
(especially SLE and rheumatoid arthritis). Another limitation of ELISA is that many patients with
early disease test negative; for example, up to 79% of patients with erythema chronica migrans
(ECM) have negative ELISA. Therefore, patients with ECM should not undergo serological
testing (a negative test would therefore have an impact on the overall management); actually,
ECM in someone from an endemic area with suggestive clinical history is diagnostic of Lyme.

Q8)

eu

ro

a. Glucocorticoids-induced cataract.
Chloroquine-retinopathy.
b. Left olecranon bursitis.
Rheumatoid nodules on his left forearm extensor surface.
c. Feltys syndrome.
Secondary AA amyloidosis.
d. Cervical cord compression.
The plain X-ray film shows many deformities of rheumatoid arthritis besides bony ankylosis.
Few notes about hand involvement in rheumatoid arthritis:
1. Up to 5% of patients will develop carpal tunnel syndrome with hand pain and paresthesia
and may patients wrongly attribute this to their fingers arthritis.
2. Testing for hand grip strength is a sensitive indicator of early disease and a useful
parameter of disease activity and progression.
3. Very acute inflammation may result in whole hand swelling and even pitting edema on
the dorsum of the hands giving rise to boxers gloves.
[70]

www.Neurology4MRCP.com

Dr.Osama S.M Amin MRCPI MRCPS(Glasg)

MRCPI Part II Written Mock Test

Q9)
a. Primary sclerosing cholangitis.
Metastatic colonic cancer in the liver.
b. Liver function tests.
Abdominal ultrasound.
Colonoscopy.
ERCP.
Jaundice in ulcerative colitis can be due to many causes. Dont forget associated immune
hemolytic anemia and drug-induced hemolysis.
Patients with primary sclerosing cholangitis may have the following:

Cholestasis and its consequences: jaundice, itching, fat-soluble vitamin deficiency...etc.


Ascending cholangitis; fever rigor, septicemia...etc.
Biliary stones.
Cholangiocarcinoma.
Colonic cancer in patients with ulcerative colitis.

P.
co

1.
2.
3.
4.
5.

4M
R

Note that:

eu

ro

lo
gy

1. The majority of patients of PSC are asymptomatic or minimally symptomatic at the time
of diagnosis.
2. PSC should always be considered in ulcerative colitis patients with otherwise
unexplained liver function tests (especially raised alkaline phosphatase).
3. The diagnosis is secured by finding multifocal intra- and extra-hepatic biliary ductal
stricture and dilatation; about 11% of patients have involvement of the intrahepatic
biliary system while 2% only have isolated involvement of extrahepatic ducts). The gall
bladder and its cystic duct might well be involved as well.
4. The disease is progressive and death usually occurs after an average of 12 years from
diagnosis. Liver failure is the usual cause of death.

Q10)
a. Church-Strauss syndrome.
b. Congestive heart failure.
Constrictive pericarditis.
c. Normochromic normocytic anemia.
Leukocytosis.
Neutrophilia.
Eosinophilia.
d. High dose glucocorticoids.

[71]

www.Neurology4MRCP.com

Dr.Osama S.M Amin MRCPI MRCPS(Glasg)

MRCPI Part II Written Mock Test

The disease usually results in rapidly progressive glomerulonephritis rather than nephrotic
syndrome. Cyclophsophamide, IVIG, and plasma exchange can be used in severe and or
refractory disease.
Asthma is one of the cardinal features of Church-Strauss syndrome (CSS) and 8-10 years may
pass before the diagnosis of CSS is made. These asthmas are usually chronic and severe so as to
necessitate the use of long-term glucocorticoid therapy. As the full vasculitic phase approaches,
the number and severity of asthma exacerbations are increased.
Serum rheumatoid factor is usually positive at a low titer rather than being negative. There are
polyclonal hypergmmaglobulinemia, circulating immune complexes, and elevated serum IgE
(which may be very high, but usually fluctuates with the activity of the vasculitic phase).

eu

ro

lo
gy

4M
R

P.
co

Death usually occurs in the vasculitic phase and has many causes. Cardiac failure and/or
myocardial infarction ranks the first on the list to be followed by cerebral hemorrhage, renal
failure, GIT hemorrhage, and finally status asthmaticus.

[72]

www.Neurology4MRCP.com

Dr.Osama S.M Amin MRCPI MRCPS(Glasg)

MRCPI Part II Written Mock Test

Data Interpretation/Answers
Q1)
a. Elevated left atrial pressure.
Elevated transmitral diastolic gradient.
Elevated pulmonary artery systolic and diastolic pressures.
Elevated right ventricular systolic and diastolic pressures.
(and the mean right atrial pressure is elevated).
b. Severe mitral stenosis with severe secondary pulmonary hypertension.

4M
R

P.
co

The symptoms of mitral stenosis are primarily related to its severity as judged by parameters such
as left atrial pressure, transmitral diastolic gradient, pulmonary hypertension, and cardiac index.
Actually, ,many patients deny symptoms as the progression in the disease is matched by gradually
unnoticed reduction in physical activity. Careful questioning would uncover progressive decline
in exercise tolerance. Note that embolic phenomena might be the presenting feature in some
patient with mitral stenosis, usually cerebral; previous studies documented up 30% embolic
complications before the widespread use of anticoagulation. Pulmonary hypertension is
potentially completely reversible in many patients with mitral stenosis upon intervention.

Q2)

lo
gy

a. Reduced FEV1 and reduced FVC in a restrictive pattern with reduced DLCO and normal
KCO.
b. Diffuse bilateral pleural thickening.

eu

ro

The restrictive pulmonary parameters with reduced DLCO would suggest a pulmonary
parenchymal disorder, like idiopathic pulmonary fibrosis; however, when the DLCO is corrected
for the lung volume, the KCO becomes normal. This would indicate an extra-parenchymal
restriction and the differential diagnoses are neuromuscular disorders (myasthenia gravis, polio,
MNDetc), thoracic cage rigidity (ankylosing spondylitis), and diffuse bilateral pleural
thickening (the past history of hemothoraces puts the patient at risk of the latter development).
The finding of reduced DLCO is useful at detecting early (preclinical) or mild interstitial lung
diseases in at risk patients like stage I sarcoidosis, radiation lung, drug-induced lung toxicity,
hypersensitivity pneumonitis, HIV patients (e.g. those at risk of P.carinii pneumonia), and lung,
renal or bone marrow transplant recipients.
Reduced DLCO with normal spirometry is seen in the following: early or mild pulmonary
vascular diseases (primary pulmonary hypertension, thromboembolic pulmonary hypertension,
and pulmonary vascular involvement in connective tissue diseases), early interstitial lung diseases
(before the FVC falls below the low normal range), anemia, and elevated blood
carboxyhemoglobin level.

[73]

www.Neurology4MRCP.com

Dr.Osama S.M Amin MRCPI MRCPS(Glasg)

MRCPI Part II Written Mock Test

Increased DLCO, usually in excess of 140% of predictive value, is seen in morbid obesity,
exercise (before doing the test), polycythemia, asthma (a small percentage only), mild left-sided
heart failure (increased pulmonary capillary blood volume), left to right cardiac shunts, and
intrapulmonary hemorrhage.

Q3)
Celiac disease.
Selective IgA deficiency.
Gluten-free diet.
Poor dietary compliance.
Folic acid deficiency and to a lesser extent vitamin B12 deficiency.

a.
b.
c.
d.
e.

lo
gy

4M
R

P.
co

IgA anti-endomysial antibodies have moderate sensitivity and high specificity for untreated celiac
disease. With proper dietary management, these antibodies gradually fall and may eventually
become negative in properly treated patients. In patients with co-existent selective IgA deficiency
(2-5% of celiac), this antibody testing becomes falsely negative; in these cases, testing for IgG
antigliadin or IgG anti-transglutaminase antibodies is useful. Note that there is a dimorphic blood
picture in our patient. Folate deficiency is much more common than vitamin B12 deficiency as a
cause of macrocytosis (and megaloblastic anemia). The commonest cause of treatment failure in
celiacs is poor compliance with gluten-free diet.

Q4)

eu

ro

a. Serum ACTH.
Serum growth hormone.
b. Brain MRI with or without contrast.
c. The TSH is the last one to fall in pituitary failure.
Pituitary secretion of bioactive substances that interact with serum TSH assay.
This man has hypogonadotrophic hypogonadism. The presence of headache and marginally
elevated serum prolactine are well pointing towards a pituitary/hypothalamic mass lesion with
pituitary stalk damage. Serum TSH is the last to fall in panhypopituitarism; in addition, the failed
pituitary may secrete bioactive substances (instead of whole TSH) that falsely interact with the
TSH bioassay. Static or dynamic testing for the rest of pituitary hormones is indicated to assess
the pituitary reserve in this patient. The elevation in serum prolactine in this man is not sufficient
to secondarily reduce the FSH/LH secretion and is likely representing disconnection
hyperprolactinemia.

[74]

www.Neurology4MRCP.com

Dr.Osama S.M Amin MRCPI MRCPS(Glasg)

MRCPI Part II Written Mock Test

Q5)
a. Increased anion gap metabolic acidosis with increased osmolal gap, mild renal
impairment, and normoglycemia.
b. Serum ethylene glycol.
c. Intravenous fomepizole.
Hemodialysis.
The above findings together with hematuria and prominent oxalate renal crystals might well be
explained by ethylene glycol intoxication (visual impairment goes with methanol poisoning).
General lines of management in our patient are:

P.
co

1. ABC.
2. Sodium bicarbonate infusion for severe systemic acidosis.
3. LDH inhibitors: intravenous fomepizole (preferred) or ethanol (when the former is
unavailable).
4. Hemodialysis in severe intoxication, evidence of very high blood levels, or in those with
end organ toxicity.
5. Cofactor treatment with pyridoxine, folate, or thiamine to optimize nontoxic pathways of
toxin (or its metabolites) elimination.

lo
gy

4M
R

Note that activated charcoal is useless for GIT decontamination in patients with ethylene glycol
(or methanol) intoxication. Clinically, measuring the osmolal gap cannot distinguish among
ethanol, isopropyl alcohol, methanol, or ethylene glycol intoxication (the overall clinical picture
is useful).
The patients lab findings argue against diabetic ketoacidosis, established renal failure (acute or
chronic) and ethanol intoxication.

Q6)

eu

ro

Calculated Sosm = 2 x serum Na + [glucose]/18 + BUN/2.8

a. Respiratory muscles weakness.


Pulmonary thromboembolism.
Pulmonary aspiration.
b. Need for assisted ventilation.
Preceding Campylobacter infection.
Presence of axonopathy (primary or secondary).
The clinical picture and the CSF profile are consistent with Guillain-Barr syndrome. The
mortality figure is 4-5% and death can result from respiratory weakness, sudden cardiorespiratroy
arrest (dysautonomia), massive pulmonary embolism, severe pulmonary aspiration, and
septicemia from bed sores (or UTI or canula-associated infection).

[75]

www.Neurology4MRCP.com

Dr.Osama S.M Amin MRCPI MRCPS(Glasg)

MRCPI Part II Written Mock Test

Respiratory failure can ensue rapidly and portends a poor prognosis. It usually occurs in 20-30%
of cases. Parameters of impending respiratory arrest (and the need for mechanical ventilation) are
FVC < 20 ml/Kg, maximum inspiratory pressure <30 cm H2O, and maximum expiratory pressure
< 40 cm H2O. Patients with at least 6 out of the following have >85% risk of developing
respiratory failure: onset to hospital admission less than a week, inability to stand, inability to lift
the elbow, inability to lift the head, inability to cough, and raised liver transaminases.

Q7)

P.
co

a. Hemolytic anemia.
Gilberts syndrome.
b. Complete blood count.
Blood film.
Intravenous nicontinic acid test.
Serum bilirubin following low lipid 300 kcal diet.
(You may also write down reticulocyte count, osmotic fragility test, hemoglobin
electrophoresis, serum LDHetc i.e. tests for hemolysis, but at least one investigation
must also be for Gilberts s as to cover both).

lo
gy

4M
R

Note that specific provocative tests for Gilberts are seldom used in clinical practice. The
commonest cause of isolated indirect hyperbilirubinemia is hemolysis, not Gilberts. No specific
treatment is available for the latter; the most important aspect in the management is reassurance
and education about the benign nature of the disease (however, some patient may develop higher
incidence of adverse drug effects, especially with irinotecan).

ro

Q8)

eu

a. Systemic lupus erythematosus.


b. Shrinkage lung syndrome.
Interstitial pulmonary fibrosis.
Cardiac failure.
(and pulmonary thromboembolism, pleural effusion, pulmonary hypertension, alveolar
hemorrhage, and acute lupus pneumonitis).
The combination of anemia and reticulocytosis points towards hemolysis. The grossly raised ESR
with normal CRP is a well-known feature of SLE (unless there is co-existent infection or serositis
which would raise serum CRP). She has nephritic picture (note the active urinary sediment). Her
ECG is that of acute pericarditis.

[76]

www.Neurology4MRCP.com

Dr.Osama S.M Amin MRCPI MRCPS(Glasg)

MRCPI Part II Written Mock Test

Q9)
a. Sputum for acid fast bacilli.
Sputum culture using Lowenstein-Jensen medium.
Pleural aspirate.
b. Brain CT scan with contrast.
CSF analysis
c. Anti-tuberculous chemotherapy with adjunctive glucocorticoid therapy.

4M
R

P.
co

The patients clinical scenario has 2 differential diagnoses in general; tuberculosis and systemic
vasculitis. The latter would be unusual without neutrophilic leukocytosis (the absence of renal
involvement does not refute the diagnosis of vasculitis). Serum albumin is low as part of negative
phase responses. Although the chest X-ray site of the opacity is uncommon for tuberculosis, the
associated high ESR, normochromic anemia, normal WBCs, with the patients ethnic
background, should always prompt the physician search for tuberculous infection. Headache with
6th cranial nerve palsy might well indicate tuberculous meningitis. Brain CT scan with contrast
can show granular ependymitis, diffuse meningeal enhancement, intracerebral tuberculoma(s), or
hydrocephalus. Around 30-50% of patients with tuberculous meningitis have negative skin
tuberculin testing. The slightly low serum potassium may represent SIADH (both, the chest and
the CNS infections can produce this).

Q10)

ro

lo
gy

a. No. Dermatitis herpetiformis is an intensely itchy skin condition.


b. Coarse granular IgA deposition in the papillary dermis.
c. The non-itch is because the patient is receiving treatment (gluten-free diet with dapsone).
The non-vesicular nature is that itching has ruptured the vesicles.

eu

Celiac disease (note his anemia and hypoalbuminemia) is associated with the following skin
conditions: dermatitis herpetiformis (DH), acquired icthyosis, cutaneous vasculitis, cutaneous
amyloid, nodular prurigo, eczema, epidermal necrolysis, pustular dermatitis, and pityriasis rubra
pillaris.
DH is an itchy papulovesicular eruption that is seen symmetrically on the elbows, knees,
buttocks, face, trunketc. The predominant symptom is itching and skin burning that is rapidly
relieved by rupture of the vesicles. We may see only papules, erosions (from ruptured vesicles),
and excoriation marks.
The patient was given the diagnosis already and must have been given a trial of dapsone with an
advice about gluten free diet.

[77]

www.Neurology4MRCP.com

This page is left blank

[78]

MRCP(I) Part I Mock

Dr.Osama SM Amin MRCP(I) MRCPS(Glasg)

www.Neurology4MRCP.com

56

Das könnte Ihnen auch gefallen